Mmwnon Stem

You might also like

Download as pdf or txt
Download as pdf or txt
You are on page 1of 108

P U P

Instructional Materials in
GEED 10053
Mathematics in the Modern World

compiled by

DMS Faculty

College of Science
Polytechnic University of the Philippines

2020
for the sole noncommercial use of the
Faculty of the Department of Mathematics and Statistics
Polytechnic University of the Philippines

2020

Conributors:

Abdul, Alsafat
Atienza, Jacky Boy
Bang-as, Pamela
Bernardino, Rhea
Cabanig, Sarah Jean
Criseno, Regine
Dilla, Perlyn Mae
Duarte, Rafael
Elizon, Katrina
Equiza, Cynthia
Hernandez, Andrew
Isaac, Emelita
Lara, Jose Alejandro Constantino
Longhas, Paul Ryan
Macatangay, Shaina Lyra
Malvar, Rolan
Nuguid, Kenneth James
Saguindan, Ian
Sta. Maria, John Patrick
Republic of the Philippines
POLYTECHNIC UNIVERSITY OF THE PHILIPPINES
COLLEGE OF SCIENCE
Department of Mathematics and Statistics

Course Title : MATHEMATICS IN THE MODERN WORLD

Course Code : GEED 10053

Course Credit : 3 units

Pre-Requisite : GENERAL MATHEMATICS, STATISTICS AND PROBABILITY (SHS)

Course Description :

The course deals with the nature of mathematics, appreciation of its practical, intellectual and
aesthetic dimensions, and application of mathematical tools in daily life. It also bridges the study of
mathematics to other domains of interest like business, finance, social sciences and arts and design.

COURSE LEARNING PLAN

Week Dates Topics and Subtopics

I. Nature of Mathematics

Mathematics in Nature

Week 1 10/5 – 10/11 1. Patterns and Numbers in Nature


2. Fibonacci Sequence
3. Mathematics for Our World

Language of Mathematics
10/12 – 10/18
Week 2-3
10/19 – 10/25 1. Propositions and Logical Connectives
2. Sets, Operations and Venn Diagrams

Problem Solving
10/26 – 11/1
Week 4-5 1. Inductive and Deductive Reasoning
11/2 – 11/8
2. Polya’s Guidelines for Problem Solving
3. Mathematical Problems involving Patterns

II. Mathematics as a Tool: Statistics and Data Management

Week 6 11/9 – 11/15 Data Gatheing and Sampling Techniques


1. Steps in Statistical Investigation
2. Sampling Techniques, Sample Size Considerations, Methods of
Data Collection
3. Levels of Measurement

Data Presentation

Week 7 11/16 – 11/22 1. Tabular Presentations: Frequency Distributions and


Crosstabulations
2. Graphical Presentations: Graphs, Charts, Time Series Plots

Descriptive Measures

Week 8 11/23 – 11/29 1. Measures of Central Tendency


2. Measures of Dispersion or Variability

III. Special Topics

Financial Mathematics

1. Simple and Compound Interest


Week 9-11 11/30– 12/20 2. Ordinary Annuities
3. Paying Off a Debt or Loan Repayment
4. Other Applications of Financial Mathematics

Mathematics of Voting and Apportionment


Week 12-14 1/4– 1/24
1. Voting Methods
2. Apportionment

FINAL ASSESSMENT

*Note: Financial Mathematics and Mathematics of Voting and Apportionment are required special topics for the programs under
the following colleges: Accountancy and Finance (CAF), Arts and Letters (CAL), Business Administration (CBA), Communication
(COC), Education (CoED), Human Kinetics (CHK), Political Science and Public Administration (CPSPA), Social Sciences and
Development (CSSD) and Tourism, Hospitality and Transportation Management (CTHTM).

COURSE GRADING SYSTEM

The final grade will be based on the weighted average of the student’s scores on each test assigned at
the end of each lesson. The final SIS grade equivalent will be based on the following table according to
the approved University Student Handbook.

Class Standing (CS) = (Weighted Average of all the Chapter/Unit Tests ÷ 2) + 50%
Midterm and/or Final Exam (MFE) = (Weighted Average of the Midterm and/or FinalTests ÷ 2) + 50%

Final Grade = (70% x CS) + (30% x MFE)


SIS Grade Percentage/Equivalent Description
1.00 97.00 - 100 Excellent
1.25 94.00-96.99 Excellent
1.50 91.00-93.99 Very Good
1.75 88.00-90.99 Very Good
2.00 85.00-87.99 Good
2.25 82.00-84.99 Good
2.50 79.00-81.99 Satisfactory
2.75 77.00-78.99 Satisfactory
3.00 75.00-76.99 Passing
5.00 65.00-74.99 Failure
Inc Incomplete
W Withdrawn
Final grades are rounded off to 2 decimal places.

Reference Materials:

• Smith, Karl J. The Nature of Mathematics. 12ed. Cengage Learning. 2012


• Angel, Abbott, Runde. Survey of Mathematics with Applications. 10ed. Pearson. 2016
• Lippman, David. Mathematics in Society. 2ed. 2017
• Thomas, Christopher. Schaum’s Outline of Mathematics for the Liberal Arts. McGrawHill. 2009

Prepared by: Noted by:

Kenneth James T. Nuguid/ Ian J. Saguindan Edcon B. Baccay


Faculty Members Chairperson
Department of Mathematics and Statistics Department of Mathematics and Statistics

Approved by:

Dr. Lincoln A. Bautista


Dean, College of Science

Dr. Emanuel C. de Guzman


Vice President for Academic Affairs
Contents

S
DM
1 Mathematics in Our World . . . . . . . . . . . . . . . . . . . . . . . . . . . . . . . . . 5
1.1 Overview: What is mathematics? . . . . . . . . . . . . . . . . . . . . . . . . . 5
1.2 Patterns and Numbers in Nature . . . . . . . . . . . . . . . . . . . . . . . . . 5
1.3 Fibonacci Sequence . . . . . . . . . . . . . . . . . . . . . . . . . . . . . . . . 12
1.4 Mathematics for Our World . . . . . . . . . . . . . . . . . . . . . . . . . . . . 17
2 Logic and Sets . . . . . . . . . . . . . . . . . . . . . . . . . . . . . . . . . . . . . . . 22
2.1 Propositions . . . . . . . . . . . . . . . . . . . . . . . . . . . . . . . . . . . . 22
2.2 Compound Propositions . . . . . . . . . . . . . . . . . . . . . . . . . . . . . . 23
2.3 Sets . . . . . . . . . . . . . . . . . . . . . . . . . . . . . . . . . . . . . . . . 28
3 Problem Solving . . . . . . . . . . . . . . . . . . . . . . . . . . . . . . . . . . . . . . 34
P
3.1 Inductive and Deductive Reasoning . . . . . . . . . . . . . . . . . . . . . . . . 34
3.2 George Polya’s Guidelines for Problem Solving . . . . . . . . . . . . . . . . . . 37
4 Statistics and Data Management . . . . . . . . . . . . . . . . . . . . . . . . . . . . . 41
PU

4.1 Basic Concepts . . . . . . . . . . . . . . . . . . . . . . . . . . . . . . . . . . . 41


4.2 Steps in Statistical Investigation . . . . . . . . . . . . . . . . . . . . . . . . . . 45
4.3 Sampling and Sampling Techniques . . . . . . . . . . . . . . . . . . . . . . . . 45
4.4 Sample Size Considerations . . . . . . . . . . . . . . . . . . . . . . . . . . . . 46
4.5 Methods of Data Collection . . . . . . . . . . . . . . . . . . . . . . . . . . . . 49
4.6 Levels of Measurement . . . . . . . . . . . . . . . . . . . . . . . . . . . . . . . 49
4.7 Presentation of Data . . . . . . . . . . . . . . . . . . . . . . . . . . . . . . . . 50
4.8 Measures of Central Tendency . . . . . . . . . . . . . . . . . . . . . . . . . . . 54
4.9 Measures of Dispersion or Variability . . . . . . . . . . . . . . . . . . . . . . . 58
5 Financial Mathematics . . . . . . . . . . . . . . . . . . . . . . . . . . . . . . . . . . . 64
5.1 Simple and Compound Interest . . . . . . . . . . . . . . . . . . . . . . . . . . 64
5.2 Ordinary Annuities . . . . . . . . . . . . . . . . . . . . . . . . . . . . . . . . . 69
5.3 Loan Repayment or Paying Off a Debt . . . . . . . . . . . . . . . . . . . . . . 72
Lesson 0 4

5.4 Other Applications of Financial Mathematics . . . . . . . . . . . . . . . . . . . 78


6 Voting Methods and Apportionment . . . . . . . . . . . . . . . . . . . . . . . . . . . . 83
6.1 Voting Methods . . . . . . . . . . . . . . . . . . . . . . . . . . . . . . . . . . 83
6.2 Apportionment . . . . . . . . . . . . . . . . . . . . . . . . . . . . . . . . . . . 93

S
DM
P
PU

All Rights Reserved. 2020 Abdul, Atienza, et. al.


Lesson 1 5

Lesson 1: Mathematics in Our World

Learning Outcomes
At the end of the lesson, the students are able to:

1. identify patterns in nature in the world;

2. articulate the importance of mathematics in one’s life;

3. argue about the nature of mathematics, what it is how it is expressed, represented and used;

S
4. enumerate and discuss the role of mathematics in various disciplines;

5. express appreciation for mathematics as a human endeavor.

1.1

DM
Overview: What is mathematics?
Mathematics can be defined in many ways. For some people, Mathematics is just the study of numbers.
For others, it is a set of problem-solving tools, a language, a process of thinking, and a study of patterns
among others. Whatever point of view is taken, there is no denying the reality that mathematics is
everywhere. Individuals from around the world use math in their daily lives. Mathematics has various
applications in the world. However, Mathematics is not only concerned with everyday problems, but also
with using imagination, intuition and reasoning to find new ideas and to solve puzzling problems. Math-
P
ematics is a branch of science, which deals with numbers and their operations. It involves calculation,
computation, solving of problems etc. Its dictionary meaning states that, ‘Mathematics is the science
of numbers and space’ or ‘Mathematics is the science of measurement, quantity and magnitude.’ It is
exact, precise, systematic and a logical subject.
PU

Mathematics helps us to organize and systemize our ideas about patterns; in so doing, not only can we
admire and enjoy these patterns, we can also use them to infer some of the underlying principles that
govern the world of nature.

In this lesson, attention will be focused on the nature of mathematics, patterns and numbers in nature
and the world and the uses of mathematics.

1.2 Patterns and Numbers in Nature


What are patterns anyway? We usually think of it as anything that repeats again and again. A pattern is
an arrangement which helps observers anticipate what they might see or what happens next. A pattern
also shows what may have come before. A pattern organizes information so that it becomes more useful.

All Rights Reserved. 2020 Abdul, Atienza, et. al.


Lesson 1 6

The human mind is programmed to make sense of data or to bring order where there is disorder. It seeks
to discover relationships and connections between seemingly unrelated bits of information. In doing so,
it sees patterns.

According to the National Council of Teachers of Mathematics


(1991) defines the nature of mathematics as follows: Mathe-
matics is a study of patterns and relationship, a way of thinking,
an art, a language, and a tool. It is about patterns and rela-
tionships. Numbers are just a way to express those patterns

S
and relationships. patterns

DM Patterns are everywhere. They are deeply embedded all around


us. You can observe patterns- things like colors, shapes, ac-
tions, line or curves of building, pathways or even in the gro-
cery store where boxes of various items are lined up. Number
patterns such as 2,4,6,8 and 5,10,15,20 are among the first
patterns encountered in younger years.

As we advance, we encounter more patterns and discover that number patterns are not restricted to a few
types. They could be ascending, descending, multiples of a certain number. We learned patterns through
P
the concept of functions and sequences like arithmetic and geometric sequences. Number patterns, logic
patterns, geometric patterns and word patterns are examples of the various patterns we learned in school.
However, patterns are not limited to these types. One can observe patterns in nature, art, architecture,
PU

human behavior, anywhere. On this section, we will discuss the different patterns in nature, arts and
architecture.

Patterns in nature are visible regularities of form found in the natural world. These patterns recur in
different contexts and can sometimes be modeled mathematically. Natural patterns include symmetries,
fractals, spirals, meanders, waves, foams, tessellations, cracks, and stripes. Studying patterns allows one
to watch, guess, create, and discover. The present mathematics is considerably more than arithmetic,
algebra, and geometry. The method of doing it has advanced from simply performing computations or
derivations into observing patterns, testing guesses, and evaluating results.

Let us focus on the different types of symmetric patterns, analyze and observe the similarities as well
as the differences and give examples of these types of patterns as seen in nature, arts, architecture and
mathematics.

All Rights Reserved. 2020 Abdul, Atienza, et. al.


Lesson 1 7

Symmetry

When we think of patterns, we usually think of it as something that repeats again and again. The math
of symmetry can describe what this repetition may look like and as well as why some objects seem more
orderly and organized than others. That is why we can say symmetry is the fundamental “language” of
patterns.

What is symmetry? Can you give examples of objects that are symmetric? Why do you consider them
symmetric? Is it because of balanced proportions? Or is it because you can rotate, translate or reflect
and they still look the same?

S
DM
Symmetry can be found everywhere. It can be seen from different viewpoints namely; nature, the arts and
P
architecture, mathematics; especially geometry and science. Symmetry occurs when there is congruence
in dimensions, due proportions and arrangement. It provides a sense of harmony and balance. In fact,
symmetry is one of the foremost predominant themes in arts, design and architecture all over the world
and throughout human history. Mathematical symmetry can also be explained as the passage of time, a
PU

spatial relationship and an aesthetic element found within abstract objects, theoretic models, language,
music and even knowledge itself.

Reflection or Bilateral Symmetry

Bilateral or reflection symmetry is the simplest kind of symmetry. It is one of the most common
kinds of symmetry that we see in the natural world. It can also be called mirror symmetry because an
object with this symmetry looks unchanged if a mirror passes through its middle. In other words, the
objects have a left side and a right side that are mirror images of each other. If a shape can be folded
in half so that one half fits exactly on top of the other, then we say that the shapes are symmetric. The
fold is called a line of symmetry because it divides the shape into two equal parts. Bilateral-symmetric
objects have at least one line or axis of symmetry. The lines of symmetry may be in any direction.

All Rights Reserved. 2020 Abdul, Atienza, et. al.


Lesson 1 8

S
Radial Symmetry
DM
Images Exhibiting Bilateral Symmetry

Radial symmetry is rotational symmetry around a fixed point known as the center. Images with more than
one lines of symmetry meeting at a common point exhibits a radial symmetry. An equilateral triangle
and circles are examples. You can cut along three different axes on the equilateral triangle while a circle
P
can be cut along an infinite number of axes. Consider the photo below. It has rotational symmetry. How
many lines of symmetry are possible?
PU

Radial symmetry can be found both in natural and human made objects. The photos below are examples
of rotational symmetry that can be found in the world around us.

All Rights Reserved. 2020 Abdul, Atienza, et. al.


Lesson 1 9

S
Did you know that there are other classifications of symmetric patterns. Patterns in the plane are usually
divided into three groups. These are rosette patterns (those that repeat in no direction), frieze patterns

overlaps, like wallpaper patterns.

Rosette Patterns
DM
( those that repeat in exactly one direction) and wallpaper patterns (those that repeat in more than
one direction). Let us define, discuss and identify examples of these patterns from nature and the arts.
Included in the discussion is what we call tessellations which completely cover a plane without gaps or

Rosette patterns consist of taking motif or an element and rotating and/or reflecting that element.
There are two types of rosette patterns namely cyclic and dihedral. A rosette pattern is cyclic if it
only admits rotational symmetries. On the other hand, a rosette pattern is dihedral if it admits both
P
rotational symmetries and bilateral or reflectional symmetries. The figures below exhibit rosette patterns.
Can you identify which of them are cyclic? dihedral?
PU

Frieze Patterns

A frieze or border pattern is a pattern in which a basic motif repeats itself over and over in one
direction. It extends to the left and right in a way that the pattern can be mapped onto itself by a

All Rights Reserved. 2020 Abdul, Atienza, et. al.


Lesson 1 10

horizontal translation. We can usually find these patterns in unique places like on the walls of buildings,
fabrics, borders of rugs and tiled floor.

Mathematicians have already classified all the different types of frieze patterns. It turns out that there
are only seven types.

1. Hop. The frieze pattern only admits a translational symmetry.

S
2. Step. The frieze pattern only admits a translational and glide symmetries.

DM
3. Sidle. The frieze pattern only admits translations and vertical reflections.

4. Spinning Hop. The frieze pattern only admits translations and 180◦ rotations (half-turns).
P
PU

5. Spinning Siddle. The frieze pattern only admits translations, vertical reflections, rotations, and
glide reflections.

All Rights Reserved. 2020 Abdul, Atienza, et. al.


Lesson 1 11

6. Jump. The frieze pattern only admits translations, a horizontal reflection, and glide reflection.

7. Spinning Jump. The frieze pattern admits translations, vertical reflections, horizontal reflections,
rotations, and glide reflections.

S
Mathematician John B. Conway invented the names of these frieze patterns.

Wallpaper Patterns

DM
A wallpaper pattern is a pattern with translation symmetry in two directions. It is, therefore, essentially
an arrangement of friezes stacked upon one another to fill the entire plane. Any particular wallpaper
pattern is made up of a combination of the following symmetries; reflection, rotation and glide reflection.
According to Nocon (2016), in order for a plane figure to be considered a wallpaper pattern, it must
have at least the basic unit, one copy by translation, and a copy of these two by translation in the second
direction. There must be at least two rows, each one of at least two units long.
P
PU

Beautiful patterns can be created by repeating geometric and artistic motifs according to the symmetry
of the wallpaper groups, as exemplified in works by M. C. Escher and in the patterns created by I. Bakshee

All Rights Reserved. 2020 Abdul, Atienza, et. al.


Lesson 1 12

in the Wolfram Language using Artlandia, illustrated above. There are 17 different wallpaper patterns.

Using intricate techniques, mathematicians were able to classify every wallpaper patterns possible. It is
shown that there are only 17 distinct types of wallpaper patterns.

S
Some Wallpaper Patterns

Tesselations

DM
A tessellation or tiling is a repeating pattern of figures that covers a plane with no gaps or overlaps.
It is just like a wallpaper group in which patterns are created by repeating a shape to fill the plane.

Tessellations can be created with translations, rotations, and reflections. Tessellations can be seen in
nature, arts and everyday life. Pavements, snake skin, turtle shell and a honeycomb are just few of many
examples of tessellation we see around us. A honeycomb is a perfect example of a natural tessellation.
It uses regular hexagons to form this natural mosaic around the surface area of the hive. Since these are
P
regular hexagons, each interior angle of each hexagon are 120 degrees, and all the angles in one of the
hexagons equal 720 degrees.
PU

Examples of Tesselations

1.3 Fibonacci Sequence


We start with 1 and another 1. Add them, we get 2. Add 1 and 2, we get 3. Add 2 and 3, we get 5.
Add 3 and 5, we get 8. If we continue repeating the process, we obtain the sequence

1; 1; 2; 3; 5; 8; 13; : : :

All Rights Reserved. 2020 Abdul, Atienza, et. al.


Lesson 1 13

which is known as the Fibonacci sequence. The Fibonacci sequence was invented by the Italian
Leonardo Pisano Bigollo (1180-1250), who is known in mathematical history by several names: Leonardo
of Pisa (Pisano means “from Pisa”) and Fibonacci (which means “son of Bonacci”).
To formally, define the Fibonacci sequence, we start by defining F1 = 1 and F2 = 1. For n > 2, we
define
Fn := Fn−1 + Fn−2 :

The sequence F1 ; F2 ; F3 ; : : : is then the Fibonacci sequence. Such a definition is called a recursive
definition because it starts by defining some initial values and defines the next term as a function of
the previous terms.

S
If we take the ratio of Fn to Fn−1 for n ≥ 1,

n
1
2
3
4
5
6
Fn
1
1
2
3
5
8
DM
Fn =Fn−1
-
1
2
1.5
1:666 : : :
1.6
n
8
9
10
11
12
13
Fn
21
34
55
89
144
233
Fn =Fn−1
1:61538 : : :
1:61904 : : :
1:61764 : : :
1:61818 : : :
1:61797 : : :
1:61805 : : :
7 13 1.625 14 377 1:618025 : : :
P
we see that as n gets larger and larger, the ratio gets closer and closer to a value denoted by ’. The
number ’ is called as the golden ratio and can be formally defined as
PU

Fn
’ := lim :
n−→∞ Fn−1

The symbol lim means ‘the limit as n approaches infinity’ which is usually studied in a calculus course.
n−→∞
It can be calculated that the exact value of ’ is

1+ 5
’= ≈ 1:6180339887 : : : :
2

1− 5
If we denote by ’ := , we can write the nth Fibonacci number explicitly using the formula
2
’n − ’n
Fn = √ :
5

This is known as the Binet Formula.

All Rights Reserved. 2020 Abdul, Atienza, et. al.


Lesson 1 14

2
3

1 1

S
DM
Do you see the Fibonacci Numbers? The red curve is known as the Fibonacci Spiral.

A rectangle whose side ratio (length:width) equals ’ is called a golden rectangle.

George Dvorsky (2013) emphasized that the Fibonacci sequence has captivated mathematicians, scien-
tists, artists and designers for centuries. It is a sequence with many interesting properties. Among these
is its visibility in nature. Most, if not all, natureâĂŹs most beautiful patterns contain Fibonacci numbers.

The Fibonacci numbers appear in nature in various places. These numbers are evident at the flower
P
head of a sunflower or daisy. Spirals are also easier to see and to count on pineapples and pine cones.
Fibonacci numbers are there on broccoli florets and flowers and on the arrangement of leaves around
stems on many plants too.
PU

• Pinecones, Speed Heads, Vegetables and Fruits


Spiral patterns curving from left and right can be seen at the array of seeds in the center of a
sunflower. The sum of these spirals when counted will be a Fibonacci number. You will get two
consecutive Fibonacci numbers if you divide the spirals into those pointed left and right. The
seed pods on a pinecone are also arranged in a spiral pattern. Each cone consists of a pair of
spirals, each one spiraling upwards in opposing directions. Spiral patterns can also be deciphered
in cauliflower and pineapples. Fibonacci sequence appears on these fruits and vegetables.

• Flowers and Branches


Most flowers express the Fibonacci sequence if you count the number of petals on these flowers.
For example, lilies and irises have three petals, roses and buttercups have five, delphiniums have
eight petals and so on. Some plants also exhibit the Fibonacci sequence in their growth points, on
the places where tree branches form or split. A trunk grows until it produces a branch, resulting

All Rights Reserved. 2020 Abdul, Atienza, et. al.


Lesson 1 15

in two growth points. The main trunk then produces another branch, resulting in three growth
points and then the trunk and the first branch produce two more growth points, bringing the total
to five as illustrated on the image below.

S
• Honeybees

DM
The family tree of a honey bee perfectly resembles the Fibonacci sequence. A honeybee colony
consists of a queen, a few drones and lots of workers. The following image below shows how the
family tree relates.
P
PU

• The Human Body


The human body has many elements that show the Fibonacci numbers and the golden ratio. Most
of your body parts follow the Fibonacci sequence and the proportions and measurements of the
human body can also be divided up in terms of the golden ratio.

• Geography, Weather and Galaxies Fibonacci numbers and the relationships between these
numbers are evident in spiral galaxies, sea wave curves and in the patterns of stream and drainages.
Weather patterns, such as hurricanes and whirlpools sometimes closely resemble the Golden Spiral.
The milky way galaxy and some other galaxies have spiral patterns. Planets of our solar system
and their orbital periods are closely related to the golden ratio.

All Rights Reserved. 2020 Abdul, Atienza, et. al.


Lesson 1 16

S
DM
The Golden Ratio and/or the Golden Spiral can also be observed in music, art, and designs. Appearing
in many architectural structures, the presence of the golden ratio provided a sense of balance and
equilibrium. Let’s take a look at a couple of examples.

• Architecture. The Great Pyramid of Giza: The Great Pyramid of Giza built around 2560 BC is
one of the earliest examples of the use of the golden ratio. The length of each side of the base
P
is 756 feet, and the height is 481 feet. So, we can find that the ratio of the vase to height is
756=481 = 1:5717:
PU

The Greek sculptor Phidias sculpted many things including the bands of sculpture that run above
the columns of the Parthenon. Other architectural structures that exhibits the Golden ratio include
the ff: Porch of Maidens, Acropolis, Athens; Chartres Cathedral; and Le Corbussier. Can you name
other structures that has the Golden Ratio?

• Arts. Mona-Lisa by Leonardo Da Vinci: It is believed that Leonardo, as a mathematician tried


to incorporate of mathematics into art. This painting seems to be made purposefully line up with

All Rights Reserved. 2020 Abdul, Atienza, et. al.


Lesson 1 17

golden rectangle.

An Old man by Leonardo Da Vinci: Leonardo Da Vinci explored


the human body involving in the ratios of the lengths of various
body parts. He called this ratio the "divine proportion" and
featured it in many of his paintings. We also have the The
Vetruvian Man (“The Man in Action”) by Leonardo Da Vinci;
Holy Family by Micahelangelo; Crucifixion by Raphael; The
sacrament of the Last Supper by Salvador Dali (1904-1989),
and many more.

S
1.4 Mathematics for Our World

DM
Mathematics is everywhere; whether it is on land, sea or air, online or on the front line, mathematics
underpins every nook and cranny of modern life. Far from a quaint subject to be forgotten upon leaving
school, it is the glue that holds our world.

Roger Bacon (1214-1294), an English Franciscan friar, philosopher, scientist and scholar of the 13th
century, once stated: “Neglect of mathematics works injury to all knowledge, since he who is ignorant
of it cannot know the other sciences or the things of the world.”

Math helps us understand or make sense of the world - and we use the world to understand math. It is
P
therefore important that we learn math contents needed to solve complex problems in a complex world;
learn the mathematical knowledge and skills we need to understand the world and make contributions
to the global community.
PU

Applications of Mathematics in Our World

Mathematics has so many uses of applications.

• Mathematics helps organize patterns and regularities in the world;

• Mathematics helps predict the behavior of nature and many phenomena;

• Mathematics helps control nature and occurrences in the world for our own good;

• Mathematics has applications in many human endeavors.

All Rights Reserved. 2020 Abdul, Atienza, et. al.


Lesson 1 18

S
DM
Mathematics helps organize patterns and regularities in the World

According to Ian Stewart (1995), we live in a universe of patterns. Human mind and culture have de-
veloped a formal system of thought for recognizing, classifying and understanding patterns. This formal
system of thought is what we know now as mathematics. We use mathematics to organize and system-
atize our thoughts and ideas about patterns and other regularities in this world. The development of
P
these new mathematical theories helped paved the way to the thorough understanding of the different
patterns in nature. Stewart (1995) also mentioned that our newfound understanding of natural order and
nature’s secret regularities is being used to steer artificial satellites to new destinations with far less fuel
PU

than anybody had thought possible, to help avoid wear on the wheels of locomotives and other rolling
stock, to improve the effectiveness of heart pacemakers, to manage forests and fisheries, even to make
more efficient dishwashers. But most important of all, it is giving us a deeper vision of the universe in
which we live, and of our own place in it. Yes, mathematics has indeed helped organize patterns and
consistencies in the world.

Mathematics helps predict the behavior of nature and many phenomena.

Mathematics is used to explain why the Sun set, where it went and why it returned because it was easier
to count these events in numbers than to put them into words. Based on historical patterns, we can make
forecasts or predictions to help us prepare for our daily activities.Formulas and other mathematical meth-
ods became a way of using numbers to show how things in nature happen, where and when it will happen.

All Rights Reserved. 2020 Abdul, Atienza, et. al.


Lesson 1 19

Earth scientists have relied in the past on statistical methods to forecast natural hazard events. However,
Benoit Mandelbrot, a professor of mathematical sciences at Yale University described how he has been
using fractals to find order within complex systems in nature, such as the natural shape of a coastline.
As a result of his research, earth scientists are taking Mandelbrot’s fractal approach one step further and
are measuring past events and making probability forecasts about the size, location, and timing of future
natural disasters.
Mathematics helps control nature and occurrences in the world for our own good.

Mathematical modelling and control theory can be used. By mathematical modeling we see the inputs
to events and their most likely outcomes. Knowing these inputs and seeing their consequences and

S
establishing their relationship defined quantitatively, we can prepare for calamities or natural disasters,
or better yet, we can probably stop them from happening.

DM
Control theory is defined as a field of applied mathematics that is relevant to the control of certain phys-
ical processes and systems. As long as human culture has existed, control has meant some kind of power
over the environment and control theory may be viewed as the science of modifying that environment,
in the physical, biological, or even social sense. Control theory played a major role in many technological
advances in the second half of the 20th century.

Mathematics has applications in many human endeavors making it indispensable.

Mathematics existed since the beginning of time, written or unwritten. Its unwritten history is carved in
P
all things found in cosmos , found in the patterns created in nature, appreciated in the juxtaposition of
the heavens and the earth, contrast between darkness and light , made sense in the harmony created not
just by a well-known orchestra but even by the rain drops falling on offshore wind-turbines. Its language,
PU

though considered by many as abstract is in fact easy to grasp when the logic and formula that govern it
are understood by the inquisitive minds of students, bakers, chemists , carpenters and appreciated by the
receptive hearts of the musicians - drummers, guitarists, pianists and composers; dance choreographers,
gymnasts and marathon runners.

Mathematics permeates every area of man’s life , leaving every man convinced of its value. As a tool,
mathematics is indispensable. It is needed by all people in honing their logical thinking and reasoning,
in making wise financial decisions - in budgeting or making both ends meet when financial resources are
scarce. It is needed in choosing the best interior and outdoor designs of houses , offices and business
sites. It is useful in determining traveling time and calculating the amount of fuel needed to get to the
destination. It is not just needed in the classrooms but also at home when doing the mundane baking
or preparing foods for breakfast , dinner or lunch; calculating steps when performing simple to complex
acrobatic stance; determining speed in a short distance or marathon run, preparing chemical solutions in

All Rights Reserved. 2020 Abdul, Atienza, et. al.


Lesson 1 20

a biological or chemical laboratory and the like. Indeed, its application and use are uncountable and the
list of uses it offers is unending.

As it is valuable and integral in the life of man, mathematics as a discipline that Introduces students
with the wide array of possibilities from honing problem-solving skills to enriching aesthetic judgment.

S
DM
P
PU

All Rights Reserved. 2020 Abdul, Atienza, et. al.


Lesson 2 21

Assessment

I. Patterns and Numbers in Nature

(1) Give five examples each of nature having reflection symmetry and radial symmetry.

(2) Compare and contrast (a) rotation and reflection; (b) translation and rotation.

(3) Which upper case letters of the English alphabet look the same after being rotated 90◦ ?
180◦ ?

S
(4) Classify the following frieze patterns based on Conway’s classification.

(a)
DM (c)

(d)

(b) (e)
P
II. Fibonacci Sequence
PU

(1) Enumerate the first twenty Fibonacci numbers.

(2) Use F40 = 63; 245; 986 and F38 = 39; 088; 169 to find the value of F39 . Show your reasoning.

(3) Using the Binet’s formula, calculate F4 .

III. Beyond the Walls (Performance Task)


Look for patterns Inside or outside of your house then take pictures of the patterns explored using
smart phones or digital camera. Explore, take photos, make list and identify what patterns can be
seen in nature inside your house, at the garden or park nearby or any part of the neighborhood.
Showcase your drawing skills by creating original paintings or pictures, poster, photo collage or vlogs
of the different patterns in nature, Fibonacci, golden ratio or the like that you have encountered
on your walk.

All Rights Reserved. 2020 Abdul, Atienza, et. al.


Lesson 2 22

Lesson 2: Logic and Sets

Learning Outcomes
At the end of the lesson, the students are able to

1. identify which are propositions and which are not;

2. construct compound propositions using logical connectives;

3. construct truth tables for propositions;

S
4. test validity of arguments

2.1 Propositions

DM
Mathematics is a language. As in any other types of language, we use sentences to communicate thoughts
and ideas. Mathematics is not an exception. We use propositions to communicate mathematical ideas
precisely.

Definition 1
A proposition is a declarative sentence that can be objectively identified as either true or false,
but not both. If a proposition is true, then its truth value is true and is denoted by T or 1;
otherwise, its truth value is false and is denoted by F or 0.
P
Example 1. Consider the following sentences.
PU

(1) Douglas MacArthur arrived in the Philippines in 1521. (5) Is that your laptop?

(2) Are you insane? (6) Basketball players are handsome.



(3) 2 is an irrational number. (7) There is life in other planets.

(4) Find all x such that xe−x = 2. (8) Welcome to the Philippines!

Immediately, we find that sentences (2), (4), (5), and (8) are not declarative sentences, so they are not
propositions.

Sentence (1) is a proposition because Douglas MacArthur either arrived in the Philippines in 1521 or
not. In fact, this proposition is false because historical records shows that Douglas MacArthur arrived in
the Philippines some time in 1900s.

All Rights Reserved. 2020 Abdul, Atienza, et. al.


Lesson 2 23

Sentence (3) is clearly a true proposition. Although statement (6) is a declarative sentence, it cannot
be considered a proposition because the meaning of the word “handsome” is subjective in nature. Unless
we could agree on an objective definition of “handsome”, then statement (6) cannot be considered a
proposition.

Finally, statement (7) is a proposition. Whether there is life or not in other planets, it doesn’t really
matter. The fact that this sentence is either true or false, and cannot be both true and false, makes it a
proposition. For this example, we still don’t have enough evidence to claim that proposition (7) is true
yet, and we don’t have a proof that it is false either. Hence, only time will tell when can we assign a
truth value for (7), but certainly, it has a truth value.

S
Symbolically, we denote propositions in this lesson using lower case letters, such as p; q; r; s; etc.

Definition 2

DM
The negation of a proposition p is the proposition which is false when p is true; and true when
p is false. The negation of p is denoted by ¬ p.

In the English language, we can simply state the negation of a proposition p by saying “It is not the case
that p.” However, there are many ways to express negations of statements grammatically by replacing
“is/are” by “is not/are not”, etc.

Example 2. Given the statements


P
p : Everyone in Visayas speaks Cebuano.
q : Today is Wednesday.
PU

The corresponding negations are

¬ p : Not everyone in Visayas speaks Cebuano.


¬ q : Today is not Wednesday.

2.2 Compound Propositions


A simple proposition is a proposition with only one subject and only one predicate. For example, the
proposition “Every cat that barks has a PhD.” is a simple proposition. The subject of this proposition
is “every cat that barks” and the predicate is “has a PhD.” In logic, we can combine simple propositions
to form compound propositions using logical connectives. Some of the most common connectives
are “or”, “and”, “but”, “unless”, etc.

All Rights Reserved. 2020 Abdul, Atienza, et. al.


Lesson 2 24

Definition 3
Let p and q be given propositions. The conjunction of p and q is the proposition “p and q”,
denoted by p ∧ q, which is true only when both p and q are true.

In other words, if one of p or q is false, then p ∧ q is false. We summarize this idea using the following
table.

p q p∧q
1 1 1
1 0 0

S
0 1 0
0 0 0

Example 3. Given the propositions


DM
Such a table is called a truth table for p ∧q. The truth table above illustrates the different combinations
of truth values for p and q and the corresponding truth value for the conjunction.

p : 3 is odd.
q : Elephants are mammals.
r : Philippines is a first world country.

We know that p and q are true and r is false. Therefore,


P
p ∧ q : 3 is odd and elephants are mammals.

is true, while
PU

p ∧ r : 3 is odd and Philippines is a first world country.

is false. For a more complicated example, the proposition

(¬ p) ∧ (¬ q) : Neither 3 is odd nor Philippines is a first world country.

is still false, since ¬ p is false.

Definition 4
Let p and q be given propositions. The disjunction of p and q is the proposition “p or q”,
denoted by p ∨ q, which is false only when both p and q are false.

In other words, if one of p or q is true (or both), then p ∨ q is true. The truth table for p ∨ q is given
below.

All Rights Reserved. 2020 Abdul, Atienza, et. al.


Lesson 2 25

p q p∨q
1 1 1
1 0 1
0 1 1
0 0 0

Example 4. Consider the statements p, q and r in the preceding example. The statement

p ∨ q : Either 3 is odd or elephants are mammals.

S
is true. Also,
p ∨ r : Either 3 is odd or Philippines is a first world country:

DM
Example 5. The proposition “Either 3 is odd or there is life in other planets.” is technically true since
the component “3 is odd.” is a true proposition. Whether the proposition “There is life in other planets.”
is true or false, the disjunction is always true.

Example 6. Construct a truth table for the compound statement p ∨ (q ∧ (¬ r )).

Solution. Since each of p, q, and r may assume two distinct truth values, then there are a total of
2 · 2 · 2 = 8 combinations, hence the truth table must contain eight rows as shown below.

p q r ¬ r q ∧ (¬ r ) p ∨ (q ∧ (¬r ))
P
1 1 1 0 0 1
1 1 0 1 1 1
1 0 1 0 0 1
PU

1 0 0 1 0 1
0 1 1 0 0 0
0 1 0 1 1 1
0 0 1 0 0 0
0 0 0 1 0 0

Definition 5
Let p and q be propositions. The conditional statement p −→ q is the proposition “If p, then
q.” is the proposition which is false only when p is true and q is false. The converse, inverse,
and contrapositive of p −→ q are the conditional statements q −→ p, (¬ p) −→ (¬ q), and
(¬ q) −→ (¬ p), respectively.

All Rights Reserved. 2020 Abdul, Atienza, et. al.


Lesson 2 26

The following is the truth table for p −→ q.

p q p −→ q
1 1 1
1 0 0
0 1 1
0 0 1

In the proposition p −→ q, the proposition p is also called as the premise and q is called as the
conclusion. From the truth table, we can see that a conditional statement is trivially true when the

S
premise is false.

Example 7. Suppose that your mother exclaims “If you don’t wash the dishes, then you don’t get

DM
money for a buffet.” In this conditional statement, the premise is “You don’t wash the dishes.” and the
conclusion is “you don’t get money for a buffet.” This statement can only false only when you don’t
wash the dishes but you still get money for the buffet.

Note that there are many ways to say p −→ q aside from “If p, then q.” Alternatively, we can say “q if
p” or “p implies q”, “p is sufficient for q” or “q is necessary for p.”

Example 8. Given the statements p : “ı is irrational.” and q : “3 is less than 2.”, then

p −→ q : If ı is irrational, then 3 is less than 2.


P
the converse of this conditional is

q −→ p : If 3 is less than 2, then ı is irrational.


PU

the inverse is
(¬ p) −→ (¬ q) : If ı is not irrational, then 3 is not less than 2.

and the contrapositive is

(¬ q) −→ (¬ p) : If 3 is not less than 2, then ı is not irrational.

If we assume that p is true and q is false (just like how they really are in mathematics), one verifies that
both p −→ q and (¬ q) −→ (¬ p) are false, while both q −→ p and (¬ p) −→ (¬ q) are true.

We like to emphasize that we write the negation of “ı is irrational” as “ı is not irrational” to emphasize
the fact that we actually don’t assume that the opposite of being irrational is being rational, unless
otherwise stated.

All Rights Reserved. 2020 Abdul, Atienza, et. al.


Lesson 2 27

Definition 6
Let p and q be propositions. The biconditional statement p ↔ q to be read as “p if and only
if q” is the proposition which is true only if both p and q are true or both p and q are false.

p q p↔q
1 1 1
1 0 0
0 1 0
0 0 1

S
Definition 7
A compound proposition is a tautology if its truth value remains true regardless of the truth values

DM
of its component propositions. On the other hand, a compound proposition is a contradiction if
its truth value remains false regardless of the truth values of its component propositions.

Example 9. The compound statement p ∨ (¬ p) is a tautology and the compound statement p ∧ (¬ p)


is a contradiction. This can be observed by looking at the truth table below.

p ¬ p p ∨ (¬ p) p ∧ (¬ p)
1 0 1 0
0 1 1 0
P
Definition 8
Let p and q be propositions (possibly compound). We say that p logically implies q, expressed
PU

as p =⇒ q, if the conditional statement p −→ q is a tautology. If p =⇒ q and q =⇒ p, we


say that p and q are logically equivalent and we write p ⇐⇒ q. A compound proposition that
is neither a tautology nor a contradiction is called a contingency.

Example 10. By constructing truth tables, show that p =⇒ p ∨ q and p ∧ q =⇒ p.

Solution.
p q p ∨ q p ∧ q p −→ (p ∨ q) (p ∧ q) −→ p
1 1 1 1 1 1
1 0 1 0 1 1
0 1 1 0 1 1
0 0 0 0 1 1

All Rights Reserved. 2020 Abdul, Atienza, et. al.


Lesson 2 28

In logic, the implication p =⇒ (p∨q) is called as the law of addition and the implication (p∧q) =⇒ p
is the law of simplification.

The following are some of the most common equivalences in logic.

Theorem 1
Let p; q; and r be propositions.

1. p ⇐⇒ q if and only if p ←→ q is a tautology.

2. p ⇐⇒ p.

S
3. p ∨ q ⇐⇒ q ∨ p and p ∧ q ⇐⇒ q ∧ p. (commutative properties)

properties)

6. De Morgan’s Laws DM
4. p ∨ (q ∨ r ) ⇐⇒ (p ∨ q) ∨ r and p ∧ (q ∧ r ) ⇐⇒ (p ∧ q) ∧ r . (associative properties)

5. p ∨ (q ∧ r ) ⇐⇒ (p ∨ q) ∧ (p ∨ r ) and p ∧ (q ∨ r ) ⇐⇒ (p ∧ q) ∨ (p ∧ r ). (distributive

(a) ¬(p ∨ q) ⇐⇒ (¬ p) ∧ (¬ q).


(b) ¬(p ∧ q) ⇐⇒ (¬ p) ∨ (¬ q)

7. p −→ q ⇐⇒ (¬ p) ∨ q.
P
8. ¬(p −→ q) ⇐⇒ p ∧ (¬ q).

9. p −→ q ⇐⇒ (¬ q) −→ (¬ p).
PU

10. p ←→ q ⇐⇒ (p −→ q) ∧ (q −→ p):

2.3 Sets
One of the basic concepts every student of mathematics must know is that of sets.
Definition 9
A set is a well-defined collection of objects called elements.

A collection is well-defined if for any given object we can objectively decide whether it is or is not in the
collection. Any object which belongs to a given set is said to be an element of or a member of the
given set.

All Rights Reserved. 2020 Abdul, Atienza, et. al.


Lesson 2 29

Example 11.

1. The collection of all letters in the English Alphabet is a set.

2. The collection of all handsome guys is not a set, because one cannot objectively identify if a given
guy is handsome or not, because the word “handsome” is subjective in nature.

Upper case letters are usually used to name sets. A set A can be commonly described in three ways,
by (a) listing (roster) method, (b) by set-builder notation or(c) by descriptive method. The listing

S
method describes the set by listing all the elements between braces and separated by commas (note:
in enumerating the elements of a certain set, each element is listed only once and the arrangement of
elements in the list is immaterial). The set-builder notation uses a variable (a symbol, usually a letter,

DM
that can represent different elements of a set), braces, and a vertical bar | that is read as "such that".
This is usually used when the elements are too many to list down. The descriptive method uses a
short verbal statement to describe the set.

Example 12. Using the roster method, the set of months in a year that ends with letter ‘y’ can be
represented by {January, February, May, July}.

Example 13. The set {2; 3; 4; 5; 6; 7; 8; 9} in set-builder notation is

{x | x is an integer greater than 1 but less than 10}:


P
If a is an element of a set A, we write a ∈ A. Otherwise, we write a ∈
= A. There are sets with no
elements. Such a set is said to be empty and we use the symbol ? to denote the empty set. A set
PU

with only one element is called a unit set or a singleton.

Example 14. The set of integers between 1 and 2 is empty, while the set of even prime numbers is a
singleton.

For future discussion, we will use the following notations:

• N for the set of natural or counting numbers (positive integers): {1; 2; 3; 4; :::}

• Z for the set of integers: {::: − 4; −3; −2; −1; 0; 1; 2; 3; :::}


a
 ff
• Q for the set of rational numbers: | a; b ∈ Z; b 6= 0
b
• R for the set of real numbers

All Rights Reserved. 2020 Abdul, Atienza, et. al.


Lesson 2 30

A set A is said to be finite if it is possible to list down all the elements of A in a list. Otherwise, A is
said to be infinite. If A is finite, the cardinality of A is the number of elements of A, which is denoted
by n(A).

Example 15. The set of all letters in the English Alphabet is finite and its cardinality is 26, because
there are 26 distinct letters in the English alphabet. On the other hand, the set of all even integers in
infinite.

Definition 10
Let A and B be sets. We say that A is a subset of B and write A ⊆ B if every element of A is

S
an element of B. We say that A and B are equal and write A = B if A ⊆ B and B ⊆ A.

Remarks.

1. For any set A, A ⊆ A and ? ⊆ A.

DM
2. If A and B are finite sets and A = B, then n(A) = n(B).

Example 16. Let A be the set of all mathematicians 20 feet high and B be the set of all PUP students.
Then A = ?. By Remark (1) above, A ⊆ B: Therefore, we can conclude that every mathematician 20
feet high is a PUP student.

Two finite sets A and B are said to be equivalent if and only if n(A) = n(B). Note that equal sets are
necessarily equivalent bu equivalent sets need not be equal.
P
Example 17. Let A = {x | x is a prime number less than 20} and B = {1; 2; 3; 4; 5; 6; 7; 8} are equiv-
alent since n(A) = 8 = n(B), however, A 6= B.
PU

Definition 11
Let A and B be sets. The union of A and B is defined as

A ∪ B = {x | x ∈ A or x ∈ B}:

The intersection of A and B is

A ∩ B = {x | x ∈ A and x ∈ B}:

Then relative complement of B in A is the set

A \ B := {x ∈ A | x ∈
= B}:

We could represent A ∪ B, A ∩ B, and A \ B in terms of Venn Diagrams as shown below.

All Rights Reserved. 2020 Abdul, Atienza, et. al.


Lesson 2 31

Example 18. Let A = {0; 1; 3; 5; 7} and B = {1; 2; 4; 7; 9}. Then A ∪ B = {0; 1; 2; 3; 4; 5; 7; 9},

S
A ∩ B = {1; 7} and A \ B = {0; 3; 5}.

DM
In most of the interesting instances in mathematics, we normally talk about a particular set of objects
at a given time. The set of all objects of interest is called as the universal set, generically denoted as
U . If A ⊂ U , the complement of A is defined as the set

A0 = U \ A = {x ∈ U ∈ x ∈
= A}:

Using the De Morgan’s Law of logic, one can readily verify that

(A ∪ B)0 = A0 ∩ B 0 and (A ∩ B)0 = A0 ∪ B 0 :


P
We can use our knowledge of sets to solve some word problems.
PU

Example 19. At a certain high school, each student is a member of the English Club, the Science
Club, or the Mathematics Club. Of the 79 students asked, 33 are members of the English Club, 37 are
members of the Math Club, and 37 are members of the of the Science club. Furthermore, 7 are members
of both the English and the Math Clubs, 12 are members of both the English and the Science Clubs,
and 9 are members of the Science and Math Clubs. No high school student is a member of all the three
clubs. How many joined only the Math Club?

Solution Let E, S, and M denote the sets of members of English, Science, and Mathematics Club,
respectively. As given in the problem, the universal set U has cardinality n(U ) = 79, n(E) = 33,
n(M) = 37, and n(S) = 37. Furthermore, n(E ∩ M) = 7, n(E ∩ S) = 12, and n(S ∩ M) = 9. The last
condition imply that E ∩ S ∩ M = ?. This situation can be represented by the following Venn diagram.

All Rights Reserved. 2020 Abdul, Atienza, et. al.


Lesson 2 32

S
Since n(E ∩ S ∩ M) = 0, then the number of students who only joined the Mathematics Club is

DM
n(M) − n(E ∩ M) − n(S ∩ M) = 37 − (7 + 9) = 21:
P
PU

All Rights Reserved. 2020 Abdul, Atienza, et. al.


Lesson 2 33

Assessment
1. Write each statement in words. Let p: The plane is on time. Let q: The sky is clear.

(a) p ∧ (¬ q)
(b) q → (p ∨ ¬p)
(c) p ↔ q

2. Construct a truth table for each proposition.

(a) [(p ∧ q) ∨ r ] ↔ [(p ∧ r ) ∨ (q ∧ r )]

S
(b) [(p ∧ r ) → (q ∧ ¬r )] → [(p ∧ q) ∨ r )]

3. Prove the De Morgan’s Laws by constructing truth tables.

B = {s, q, u, a, r, e };
C = {h, e, x, a, g, o, n, s }
Determine the following:

(a) A ∪ (B ∩ C)
DM
4. Let U := Letters in the English Alphabet = {a, b, c, . . . ,x, y, z}
A = {t, r, i, a, n, g, l, e, s}

(b) (A ∪ B)0 ∩ C
P
(c) (A ∩ C) ∪ (B ∩ C)
(d) A ∩ (C ∩ U )0
(e) n[(A ∪ B) ∩ (B ∪ C)]
PU

5. A survey of 90 customers was taken at Barnes & Noble regarding the types of books purchased.
The survey found that 44 purchased mysteries, 33 purchased science fiction, 29 purchased romance
novels, 13 purchased mysteries and science fiction, 5 purchased science fiction and romance novels,
11 purchased mysteries and romance novels, and 2 purchased all three types of books (mysteries,
science fiction, romance novels). How many of the customers surveyed purchased

(a) mysteries only?


(b) mysteries and science fiction, but not romance novels?
(c) mysteries or science fiction?
(d) romance novels or mysteries, but not science fiction?
(e) exactly two types (mysteries, science fiction, romance novels)?

All Rights Reserved. 2020 Abdul, Atienza, et. al.


Lesson 3 34

Lesson 3: Problem Solving

Learning Outcomes
At the end of the lesson, the students are able to

1. differentiate between inductive and deductive reasoning;

2. utilize inductive reasoning to form conjectures;

3. use deductive reasoning to prove a conjecture;

S
4. state the Polya’s four steps in problem solving;

5. solve mathematical problems using the Polya’s four steps.

3.1

DM
Inductive and Deductive Reasoning
Human beings are said to be rational creatures because we use reasoning to come up with sound decisions
that we have to make everyday. Reasoning is our ability to use logical thinking to come up with a
decision. There are two major types of reasoning: inductive and deductive. We first talk about inductive
reasoning.

Definition 12
P
Inductive Reasoning is the process of reasoning that arrives at a general conclusion based on
the observation of specific examples.

Normally, we use inductive reasoning when we need to come up with a general conclusion, known as
PU

a conjecture, by observing certain events or examples. Generally speaking, our conjectures could be
wrong. Examples which can negate our conjectures are called counterexamples.

Example 20. In the past 30 days, we observed that the sun has risen in the east. Using inductive
reasoning, we may conjecture that the sun will rise in the east tomorrow.

Example 21. Consider the odd numbers 3; 5; 7; and 9. If we take their squares, we see that 32 = 9,
52 = 25, 72 = 49 and 92 = 81. We can observe that the squares of the given odd numbers are all odd
as well. Using inductive reasoning, we may conjecture that the square of an odd integer is also odd.

Testing Conjectures
Logically speaking, we cannot prove a general statement from a number of specific examples unless there
are only finitely many examples and we can exhaust them. However, only one counter example can prove

All Rights Reserved. 2020 Abdul, Atienza, et. al.


Lesson 3 35

that our conjecture is false.

Example 22. Let n be a positive integer. Select n distinct points at random in the circumference of a
circle and connect every pair of points in this collection by a chord. Make a conjecture about the number
of regions in the interior of the circle made by the chords and test your conjecture.

For n = 1; 2; 3; 4; 5, we draw actual circles and count the number of regions made by the chords obtained
by connecting every pair of points.

S
DM
We summarize the number of regions in the following table.

n
no. of regions
1
1
2
2
3
4
4
8
5
16

If we observe the pattern on the number of regions, they seem to be powers of 2. In fact, for
n = 1; 2; 3; 4; 5, the number of regions in the circle is 2n−1 . It is therefore reasonable for use to
give the following conjecture.
P
Conjecture. The number of regions in the interior of the circle made by connecting every pair of points
in a set of n points in the circumference is 2n−1 .
PU

The best way to test the conjecture is to check the example for the next larger n, which is n = 6.
Constructing the circle for n = 6 and counting the regions,

we see that the number of regions is 31 and not 26−1 . This counterexample disproves our conjecture.

All Rights Reserved. 2020 Abdul, Atienza, et. al.


Lesson 3 36

Definition 13
Deductive reasoning is the process of reasoning that arrives at a conclusion based on previously
accepted general statements.

Deductive reasoning does not rely on examples. We make our conclusion based on general statements
whose truth value is known or assumed. Formal mathematics is usually based on this type of reasoning.
We first lay down definition of terms, and assume basic true statements called axioms and derive true
statements from these axioms called as theorems.

Example 23. The following are examples of deductive reasoning.

S
1. Starfish are invertebrates. Patrick is a starfish. Therefore, Patrick is invertebrate.
√ √
2. Every rational number is a real number. The number −1 is not real. Therefore, −1 is not
rational.

DM
Inductive reasoning cannot in general prove general statements as this relies on examples only. In contrast,
we can use deductive reasoning to prove a certain conjecture.

Example 24. Choose any number. Multiply by 3. Add 6 to the result. Divide the result by 3. Finally,
subtract the original number from the result of the previous step. Use inductive reasoning to make a
conjecture about the final result and use deductive reasoning to prove the conjecture.

Solution. We first consider few examples.


P
test number 9 15 28
multiply by 3: 27 45 84
PU

add 6: 33 51 90
divide by 3: 11 17 30
subtract the orig. no. 2 2 2

We see that based from the three test numbers, the final results are the same and are all equal to 2.
There is a reason to conjecture that the final result will always be 2 regardless on where we start. To
prove this claim, take an arbitrary number x.
multiply by 3: 3x
add 6: 3x + 6
3x + 6
divide by 3: =x +2
3
subtract the orig. no: (x + 2) − x = 2.
Therefore, as claimed, it is now proven that we will always end up with 2.

All Rights Reserved. 2020 Abdul, Atienza, et. al.


Lesson 3 37

3.2 George Polya’s Guidelines for Problem Solving


In 1945, mathematician George Pólya devised a model for problem solving and published it in his book
How to Solve It. The book contains a collection of mathematical problems and selected strategies on
dealing these. His problem solving model, which he called heuristic (or serving to discover), is as follows.

POLYA’S FOUR STEPS:

1. Understand the problem. Ask questions, experiment, or otherwise rephrase the question in
your own words.

S
2. Devise a plan. Find the connection between the data and the unknown. Look for patterns, relate
to a previously solved problem or a known formula, or simplify the given information to give you
an easier problem.

1. Draw a diagram.

2. Solve a simpler problem.


DM
3. Carry out the plan. Check the steps as you go.

4. Look back. Examine the solution obtained. In other words, check your answer.

Together with these guidelines, the following are some of his recommended strategies:

5. Guess and check.

6. Find a pattern.

3. Make a table. 7. Use a formula or an equation.


P
4. Work backwards. 8. Using logical reasoning.

Example 25. In a seminar, 30 attendees were present. During their meet-and-greet activity, they were
PU

asked to have a handshake with everyone in the room. If each one did handshake with everyone, how
may handshakes took place?

Solution.

Step 1. Understand the Problem. There were 30 attendees present. A simple handshake means
letting a distinct pair be recognized. Moreover, if A shakes hands with B, then B shakes hands
with A as well.

Step 2. Devise a plan. We start with solving simpler cases, say 3, 4 and 5 persons. We can draw
a diagram where a person is represented by nodes while handshakes by arcs connecting the
nodes. From here, we try to find a pattern.

Step 3. Carry out the plan. The following figures represent the handshakes that took place among
3, 4 and 5 persons.

All Rights Reserved. 2020 Abdul, Atienza, et. al.


Lesson 3 38

A group of 3 persons makes 3 handshakes, a group of 4 persons makes 6 handsakes and a


group of 5 persons makes 10 handshakes. Now, for each case with k persons, each of these
persons has to have a handshake with the other k − 1 persons. So, the product k(k − 1) is

S
the number of all handshakes from individual perspective. Note that if A shakes hands with B,
then B shakes hands with A as well. Thus, only half of k(k − 1) represents the total number of
k(k − 1)
handshakes. Hence, a pattern is generalized by will lead to the number of handshakes

DM
2
=
2
2
that took place in a group of k persons. Therefore, there were a total of

k(k − 1) 30(30 − 1)
= 435 handshakes.

Step 4. Look back. Every person will be shaking hands with 29 other. Thus, 870 handshakes are
noted for individual perperspective. Half of which is 435.
P
Example 26. Andrew has some magic cards to trade. Ian has 2 more than 2 times the number of magic
cards Andrew has. Patrick has 2 less than Ian. Ken has 4 less than 2 times the number of magic cards
PU

Patrick has. Patrick has 8 magic cards. How many magic cards does Andrew have to trade?

Solution.

Step 1. Understand the Problem. The number of magic cards Ian has depends on the number of
magic cards Andrew has. The number of magic cards Patrick has depends on the number of
magic cards Ian has. The number of magic cards Ken has depends on the number of magic
cards Patrick has. It is clear that 8 cards are in Patrick’s possession.

Step 2. Devise a plan. We can settle this by working backwards starting from the number of magic
cards Patrick has. Making a table may aid organization.

Step 3. Carry out the plan.

All Rights Reserved. 2020 Abdul, Atienza, et. al.


Lesson 3 39

Statements of Hints Arithmetic Sense Remarks


Patrick has 8 magic cards. 8 This is the last event.
Ken has 4 less than 2 times Operation is not
the number of magic cards (2 × 8) − 4 = 12 yet revered.
Patrick has. Ken has 12 cards.
Patrick has 2 less Operation is reversed.
less than Ian. 8 + 2 = 10 Ian has 10 cards.
Ian has 2 more than Operation is reversed.
2 times the number of (10 − 2)=2 = 4 Andrew has 4 cards.
magic cards Andrew has.

S
Step 4. Look back. If Andrew has 4 magic cards, then Ian has 2 more than twice of 4 or 10 magic
cards. If Ian has 10 magic cards, then Patrick has 10 - 2 = 8 magic cards. Lastly, if Ken has

DM
4 less than twice of 8 of 12 magic cards.
P
PU

All Rights Reserved. 2020 Abdul, Atienza, et. al.


Lesson 3 40

Assessment
1. Explain why you can never be sure that a conclusion you arrived at using inductive reasoning is
true.

2. Select any two-digit number. Multiply it by 9. Then add the digits. Keep adding the digits in
the answer until you get a single-digit answer. Using inductive reasoning, what can you conjecture
about any whole number multiplied by 9? Use deductive reasoning to prove that your conjecture
is true.

3. Use Polya’s Four Steps to solve the following problems.

S
(a) Susie’s age this year is a multiple of 5. Next year, her age is a multiple of 7. What is her
present age?

DM
(b) Consider a square whose side is 1 unit. If the measure of its side is doubled, what will be its
new area as compare to the smaller square? How about if the side of the smaller square was
tripled, what will be its new area?
(c) How many perfect squares are there between 1,000,000 and 9,000,000?
(d) Determine the number of different triangles that can be drawn given eight noncollinear points?
(e) There are 25 students asked by their literature instructor regarding with the type of literary
works they prefer to read. He found out that 10 prefer to read novels, 11 prefer to read short
stories, 15 prefer to read poems, 5 for both novels and short stories, 4 both short stories and
P
poems, 7 for both novels and poems, and 3 prefer all. How many students prefer none of the
given types of literary works?
PU

All Rights Reserved. 2020 Abdul, Atienza, et. al.


Lesson 4 41

Lesson 4: Statistics and Data Management

Learning Outcomes
At the end of the lesson, the students are able to

1. demonstrate the ability to apply fundamental concepts in exploratory data analysis;

2. define the field of Statistics in terms of its definition and application;

3. enumerate the procedures involved in collecting data;

S
4. distinguish between the nominal, ordinal, interval and ratio methods of data measurement;

5. recognize the various ways to present data;

DM
6. identify the features that describe a data distribution.

Statistics is the study of the collection, organization, analysis, interpretation, and presentation of data.
It deals with all aspects of data, including the planning of its collection in terms of the design of
surveys and experiments. Some consider statistics a mathematical body of science that pertains to the
collection, analysis, interpretation or explanation, and presentation of data, while others consider it a
branch of mathematics concerned with collecting and interpreting data. Because of its empirical roots
and its focus on applications, statistics is usually considered a distinct mathematical science rather than
a branch of mathematics.
P
4.1 Basic Concepts
PU

Statistics is defined as a branch of mathematics which is concerned with facilitating wise decision-
making in the face of uncertainty and that, therefore develops and utilizes techniques for collection,
effective presentation, and proper analysis of data.

Branches of Statistics

1. Descriptive Statistics is concerned with the description and summarization of data, It deals with
the techniques used in the collection, presentation, organization, and analysis of the data on hand.

2. Inferential Statistics is concerned with the drawing of conclusions from data. It deals with the
techniques used in generalizing from samples to populations, performing estimations and hypothesis
tests determining relationships among variables, and making predictions.

All Rights Reserved. 2020 Abdul, Atienza, et. al.


Lesson 4 42

Functions of Statistics

1. Condensation. Generally speaking by the verb ‘to condense’, we mean to reduce or to lessen.
Condensation is mainly applied at embracing the understanding of a huge mass of data by providing
only few observations.

2. Comparison. Classification and tabulation are the two methods that are used to condense the
data. They help us to compare data collected from different sources. Grand totals, measures
of central tendency measures of dispersion, graphs and diagrams, coefficient of correlation, etc.
provide ample scope for comparison. As statistics is an aggregate of facts and figures, comparison

S
is always possible and in fact comparison helps us to understand the data in a better way.

3. Forecasting. By the word forecasting, we mean to predict or to estimate beforehand. Given the

DM
data of the last ten years connected to the number of students enrolled in PUP, it is possible to
predict or forecast the number of students that will enroll for the near future. In business also
forecasting plays a dominant role in connection with production, sales, profits etc. The analysis of
time series and regression analysis plays an important role in forecasting.

4. Estimation. One of the main objectives of statistics is drawn inference about a population from
the analysis for the sample drawn from that population.

5. Tests of Hypothesis. A statistical hypothesis is some statement about the probability distri-
bution, characterizing a population on the basis of the information available from the sample
P
observations. In the formulation and testing of hypothesis, statistical methods are extremely use-
ful. Whether the grades of students increased because they are motivated or whether the new
teaching method is effective in discussing a particular topic are some examples of statements of
hypothesis and these are tested by proper statistical tools.
PU

Scope of Statistics

1. Statistics and Industry. Statistics is widely used in many industries. In industries, control charts
are widely used to maintain a certain quality level. In production engineering, to find whether the
product is conforming to specifications or not, statistical tools, namely inspection plans, control
charts, etc., are of extreme importance. In inspection plans we have to resort to some kind of
sampling - a very important aspect of Statistics.

2. Statistics and Commerce. Statistics are lifeblood of successful commerce. Any businessman
cannot afford to either by under stocking or having overstock of his goods. In the beginning he
estimates the demand for his goods and then takes steps to adjust with his output or purchases.
Thus statistics is indispensable in business and commerce.

All Rights Reserved. 2020 Abdul, Atienza, et. al.


Lesson 4 43

3. Statistics and Economics. Statistical methods are useful in measuring numerical changes in
complex groups and interpreting collective phenomenon. Nowadays the uses of statistics are abun-
dantly made in any economic study. Both in economic theory and practice, statistical methods
play an important role.

4. Statistics and Education. Statistics is widely used in education. Research has become a
common feature in all branches of activities. Statistics is necessary for the formulation of policies
to start new course, consideration of facilities available for new courses etc. There are many people
engaged in research work to test the past knowledge and evolve new knowledge. These are possible
only through statistics.

S
5. Statistics and Planning. Statistics is indispensable in planning. In the modern world, which can
be termed as the “world of planning”, almost all the organizations in the government are seeking

DM
the help of planning for efficient working, for the formulation of policy decisions and execution of
the same. In order to achieve the above goals, the statistical data relating to production, consump-
tion, demand, supply, prices, investments, income expenditure etc and various advanced statistical
techniques for processing, analyzing and interpreting such complex data are of importance. In
India statistics play an important role in planning, commissioning both at the central and state
government levels.

6. Statistics and Medicine. In Medical sciences, statistical tools are widely used. In order to test
the efficiency of a new drug or medicine, t - test is used or to compare the efficiency of two drugs
or two medicines, t-test for the two samples is used. More and more applications of statistics are
P
at present used in clinical investigation.

7. Statistics and Modern Applications. Recent developments in the fields of computer technol-
ogy and information technology have enabled statistics to integrate their models and thus make
PU

statistics a part of decision making procedures of many organizations. There are so many software
packages available for solving design of experiments, forecasting simulation problems etc.

Limitations of Statistics

1. Statistics is not suitable to the study of qualitative phenomenon. Since statistics is


basically a science and deals with a set of numerical data, it is applicable to the study of only
these subjects of enquiry, which can be expressed in terms of quantitative measurements. As a
matter of fact, qualitative phenomenon like honesty, poverty, beauty, intelligence etc, cannot be
expressed numerically and any statistical analysis cannot be directly applied on these qualitative
phenomenon.

2. Statistics does not study individuals. Statistics does not give any specific importance to the
individual items; in fact it deals with an aggregate of objects. Individual items, when they are taken

All Rights Reserved. 2020 Abdul, Atienza, et. al.


Lesson 4 44

individually do not constitute any statistical data and do not serve any purpose for any statistical
enquiry.

3. Statistical laws are not exact. It is well known that mathematical and physical sciences are
exact. But statistical laws are not exact and statistical laws are only approximations. Statistical
conclusions are not universally true. They are true only on an average.

4. Statistics table may be misused. Statistics must be used only by experts; otherwise, statistical
methods are the most dangerous tools on the hands of the inexpert. The use of statistical tools
by the inexperienced and untraced persons might lead to wrong conclusions.

S
5. Statistics is only one of the methods of studying a problem. Statistical method do
not provide complete solution of the problems because problems are to be studied taking the

Population and Sample DM


background of the countries culture, philosophy or religion into consideration. Thus the statistical
study should be supplemented by other evidences.

In statistics, we are often interested in gathering information from a group of objects. If the group
in consideration consists of large number of objects, we try to obtain information about the group by
examining its subgroup.
P
Definition 14
The total collection of all the elements that we are interested in is called a population. A
subgroup of the population that will be studied in detail is called a sample.
PU

In order for the data from the sample is informative about the population, it must be representative
of the population. Being representative of the population does not mean that the characteristic of the
sample is exactly that of the total population, but instead the sample was obtain in such way that every
member of the population had an equal chance to be included in the sample.

Definition 15
A sample of k members of a population is called a random sample, also called a simple random
sample, if the members are chosen in such a way that all possible choices of the k members are
equally likely.

After a random sample is obtain from the population, we can use statistical inference to draw general-
izations about the population by examining the members of the sample.

All Rights Reserved. 2020 Abdul, Atienza, et. al.


Lesson 4 45

4.2 Steps in Statistical Investigation


1. Defining the problem

(a) Identify a specific problem.


(b) Define the scope and limitations, assumptions to be made, and expected outcomes.

2. Collection of data

(a) Make sure to collect the data properly.


(b) Incomplete, fabricated, outdated, and inaccurate data are useless.

S
3. Summarization and tabulation of data

(a) This refers to organization of data in text, tables, graphs and charts, so that logical conclusion
can be derived from them.

4. Analysis of data
DM
(b) Explore the data to obtain additional insight that could contribute to the study.

(a) This pertains to the process of deriving from the given data relevant information from which
numerical descriptions can be formulated.
(b) Summarized data must be examined so that insights and meaningful information ca be pro-
duced to support decision-making or solutions to the question or problem at hand.
P
5. Interpretation of data and results

(a) Refers to the task of drawing conclusions from the analyzed data.
(b) Results must be able to answer the research problem and give recommendations.
PU

6. Presentation of the result

(a) Present all pertinent results in a clear and concise manner.


(b) Use appropriate form of media to present results.

4.3 Sampling and Sampling Techniques


Sampling refers to the process of obtaining samples from the population. Sampling maybe categorized as
either probability sampling or non-probability sampling. Probability sampling, also referred to as random
sampling, is the method of sampling in which every member of the population have equal chance of
being selected as sample; otherwise, it is considered as non-probability sampling. We should note that in
able to properly use the techniques of statistical inference, probability sampling must be used to obtain
samples.

All Rights Reserved. 2020 Abdul, Atienza, et. al.


Lesson 4 46

Probability Sampling Techniques

1. Simple Random Sampling. A probability sampling technique wherein all possible subsets con-
sisting of n elements selected from the N elements of the population have the same chances of
selection.

2. Systematic Sampling. This is a probability sampling technique wherein the selection of the
first element is at random and the selection of other elements in the sample is systematic by
subsequently taking every kth element from the random start where k is the sampling interval.

3. Stratified Random Sampling. A probability sampling method where we partition the population

S
into non-overlapping strata or group and then a proportional sample is chosen from each strata.
The actual sample is the sum of the samples derived from each strata.

DM
4. Cluster Sampling. A probability sampling technique wherein we partition the population into
non-overlapping groups or clusters consisting of one or more elements, and then select a sample
of clusters. Every member of the selected cluster will be considered as sample.

Non-Probability Sampling Techniques

1. Accidental Sampling. Sample is chosen by the researcher by the obtaining members of the
population in a convenient, often haphazard way.

2. Quota Sampling. There is specified number of persons of certain types is included in the sample.
The researcher is aware of categories within the population and draws samples from each category.
P
The size of each categorical sample is proportional to the proportion of the population that belongs
in that category.
PU

3. Purposive Sampling. The researcher employs his or her judgments on choosing which he or she
believes are representative of the population.

4. Snowball Sampling. This technique is also called referral sampling. A primary set of samples
are chosen based on the criteria set by the researcher. Information on where to find succeeding
set of sample having the same criteria will be gathered from this primary set in order to expand
the number of samples.

4.4 Sample Size Considerations


The sample size is typically denoted by n and it is always a positive integer. No exact sample size can be
mentioned here and it can vary in different research settings. However, all else being equal, large sized
sample leads to increased precision in estimates of various properties of the population.
To determine the sample size we can apply one of the following methods:

All Rights Reserved. 2020 Abdul, Atienza, et. al.


Lesson 4 47

1. Slovin’s Formula. Slovin’s formula is used to calculate the sample size n given the population
size and a margin of error E. It is a formula use to estimate sampling size of a random sample
from a given population. We can compute

N
n= ;
1 + NE 2

where N is the population size.

Example 27. A researcher plans to conduct a survey about food preference of BS Stat students. If the
population of students is 1000, use the Slovin’s formula to find the sample size if the margin of error is 5%.

S
Solution. Using the Slovin’s formula, we get

DM
n=
1000
1 + 1000(0:05)2
≈ 285:71:

Therefore, the researcher needs to survey 286 BS Stat Students.

2. Minimum Sample Size for Estimating a Population Mean. The estimated minimum sample
size n needed to estimate a population mean — to within E units at 100(1 − ¸)% confidence is

(z¸=2 )2 ff 2
n= ;
E2
where ff is the known population standard deviation, E is the margin of error and z¸=2 is a value
P
which can be obtained in the z-table.

Example 28. Suppose we want to know the average age of STEM students. We would like to be 99%
PU

confident about our results. From previous study, we know that the standard deviation for the population
is 1.3. How many students should be chosen for a survey if the margin of error is 0.2.

Solution. Find z¸=2 by looking at the z-table.

¸ = (1 − 0:99) = 0:01 =⇒ z¸=2 = z0:005 :

The closest z-score for 0:005 in the z-table is 2:58. Thus,

(2:58)2 (1:3)2
n= ≈ 281:23:
(0:2)2

which we round up to 282, since it is impossible to take a fractional observation. We need a 282 STEM
students as a sample for our study.

All Rights Reserved. 2020 Abdul, Atienza, et. al.


Lesson 4 48

3. Minimum Sample Size for Estimating a Population Proportion The estimated minimum
sample size n needed to estimate a population proportion p to within E at 100(1 − ¸)% confidence
is
(z¸=2 )2 p̂(1 − p̂)
n= :
E2
This is also called the Cochran Formula.

The dilemma here is that the formula for estimating how large a sample to take contains the
number p̂, which we know only after we have taken the sample. There are two ways out of this
dilemma.

S
• First, typically the researcher will have some idea as to the value of the population proportion

in the formula.
DM
p, hence of what the sample proportion p̂ is likely to be. For example, if last month 37% of
all voters thought that state taxes are too high, then it is likely that the proportion with that
opinion this month will not be dramatically different, and we would use the value 0.37 for p̂

• The second approach to resolving the dilemma is simply to replace p̂ in the formula by 0.5.
This is because if p̂ is large then 1 − p̂ is small, and vice versa, which limits their product to
a maximum value of 0.25, which occurs when p̂ = 0:5. This is called the most conservative
estimate, since it gives the largest possible estimate of n.
P
Example 29. Suppose we are doing a study on the inhabitants of a large town, and want to find out
how many households serve breakfast in the mornings. We don’t have much information on the subject
to begin with, so we’re going to assume that half of the families serve breakfast: this gives us maximum
PU

variability. Here, p̂ = 0:5. We want 95% confidence and at least 5% precision.

Solution. Find z¸=2 in the z-table. We have

¸ = (1 − 0:95) =⇒ z¸=2 = z0:025 :

The closest z-score for 0:025 in the z-table is 1:96. A 95% confidence level gives us Z values of 1.96,
we get
(1:96)2 (0:5)(1 − 0:5)
n= ≈ 384:16:
(0:05)2
Hence, a random sample of 385 households in our target population should enough to give us the
confidence levels we need.

All Rights Reserved. 2020 Abdul, Atienza, et. al.


Lesson 4 49

Finite Population Correction for Proportions

If the population is small then the sample size can be reduced slightly. This is because a given sample size
provides proportionately more information for a small population than a large population. The formula
is
n0
n= ;
n0 − 1
1+
N
where n0 is the Cochran’s sample size recommendation, N is the population size and n is the new adjusted
sample size.

S
Example 30. In the preceding example, if there were just 1000 households in the target population, we
would calculate
385
n= ≈ 278:18:
385 − 1

DM
1+
1000
All we need are 279 households in our sample, a substantially smaller sample size.

4.5 Methods of Data Collection


1. Survey Method. The survey is a method of collecting data on the variable of interest by asking
people questions. This may be done, by interview or by using questionnaires.

2. Observation. Observation is a method of obtaining data or information by using our primary


senses.
P
3. Experiment. Experiment is a method of collecting data where there is direct human intervention
on the conditions that may affect the values of the variable of interest.
PU

4.6 Levels of Measurement


1. The nominal level of measurement classifies data into mutually exclusive (non-overlapping)
categories in which no order or ranking can be imposed on the data.

Example: Gender (male, female), Zip Code, Color, Nationality, Political affiliation, Religious
affiliation.

2. The ordinal level of measurement classifies data into categories that can be ranked; however,
precise differences between the ranks do not exist.

Example: Grade(A,B,C,D,F), Rating Scale/Likert scale, Ranking of tennis players, Judging (First
place, second place, etc.

All Rights Reserved. 2020 Abdul, Atienza, et. al.


Lesson 4 50

3. The interval level of measurement ranks data, and precise differences between units of measure
do exist; however, there is no meaningful zero.

Example: Temperature, IQ, SAT score

4. The ratio level of measurement possesses all the characteristics of interval measurement, and
there exists a true zero. In addition, true ratios exist when the same variable is measured on two
different members of the population

Example: Height, Weight, volume, Time, Salary, Age

S
4.7 Presentation of Data

DM
After data have been collected, the researcher can now present them in the following logical methods.

1. Textual Form. Data are presented in paragraph of text. The text highlights the important figures
or results that the researcher wishes to focus on.

2. Tabular Form. Data appears in a systematic manner in rows and columns.


The following is an example of a Simple or One-Way Table.

Table 1
Frequency Distribution of the
P
Students Enrolled for the Last 6 Years
Year Frequency
2012 13,450
PU

2013 13,200
2014 15,389
2015 16,790
2016 18,900
2017 19,500
Total 97,229

All Rights Reserved. 2020 Abdul, Atienza, et. al.


Lesson 4 51

The following is an example of a Two-Way Table.

Table 2
Number of Students Enrolled for the Last 6 Years
When Grouped According to Sex

Year
Sex
2012 2013 2014 2015 2016 2017 Total
Male 5560 6095 7386 8056 7945 6451 41493
Female 7890 7105 8003 8734 10955 13049 55736

S
Total 13450 13200 15389 16790 18900 19500 97229

3. Graphical Form. Data or relationship among variables could be presented in visual form, thru

Types of Statistical Charts DM


graph or diagrams. In that manner, the reader can easily perceive what is being meant by the
figure or any trend being portrayed by the data.

(a) Bar Graph (Vertical Bar/Column Charts) is applicable for showing comparison of
amount of a variable of interest collected over time.

Simple Chart
P
PU

Grouped Column Charts

All Rights Reserved. 2020 Abdul, Atienza, et. al.


Lesson 4 52

Subdivided Column Charts

S
(b) Histogram is similar to the bar graph but the base of the rectangle has a length exactly
equal to the class width of the corresponding interval. Also, there are no spaces between
rectangles.

DM Histogram
P
(c) Pictograph is similar to the bar chart but instead of bars, we use pictures or symbols to
represent a value or an amount.
PU

Pictograph

(d) Pie Chart is a circular graph partitioned into several section, depicting relative percentage
with respect to the total distribution.

All Rights Reserved. 2020 Abdul, Atienza, et. al.


Lesson 4 53

Pie Chart

S
(e) Line Graph is a graph used to visualize data that changes continuously over time.

Simple Line Graph

DM Multiple Line Graph


P
PU

(f) Statistical Map is used to show data in geographical areas.

Statistical Map

All Rights Reserved. 2020 Abdul, Atienza, et. al.


Lesson 4 54

4.8 Measures of Central Tendency


A measure of central tendency or average is a location measure that pinpoints the center or typical
middle value of a data set. A convenient way of describing a set of data with a value that describes
the average characteristic a data set. The three common measures of central tendency are the mean,
median and mode.

Mean

Definition 16

S
Suppose that a variable x assumes values x1 ; x2 ; : : : ; xn . The arithmetic mean x of these values
is defined as n
1X x1 + x2 + · · · + xn
P
x
x=

DM
= xi = :
n n i=1 n

The (arithmetic) mean of x is obtained by adding all its observed values and dividing the sum by the
total number of observations.

Example 31. The scores of 15 students in Mathematics in the Modern World on an exam consisting
of 25 items are 25,20,18,18,17,15,15,15,14,14,13,12,12,10,10. Determine the mean score for this exam.

Solution. Let x denote the score of a random student from the sample of 15 students in Mathematics in
the Modern World. The sum of these scores is x = 228. Hence, the mean score of the 15 students is
P
P
228
P
x
x= = = 15:2:
n 15
PU

There are cases when the observations in a data set assume respective weights. In this case where the
weights are positive integers, we can call these weights as frequencies. The following gives a formula
for the weighted mean of a weighted data set.

Definition 17
Given the x values x1 ; x2 ; : : : ; xn assuming respective weights w1 ; w2 ; : : : ; wn , the weighted mean
is defined as
w1 x1 + w2 x2 + · · · + wn xn
P
wx
x= P = :
x w1 + w2 + · · · + wn

Example 32. Suppose that we are asked to get the mean of the data set 1; 1; 3; 3; 3; 3; 4; 4; 4; 6; 6; 8.

All Rights Reserved. 2020 Abdul, Atienza, et. al.


Lesson 4 55

Using the original formula for the arithmetic mean we find that

(1 + 1) + (3 + 3 + 3 + 3) + (4 + 4 + 4) + (6 + 6) + 8
x=
12
2·1+4·3+3·4+2·6+1·8
=
1+4+3+2+1
2 + 12 + 12 + 12 + 8
=
12
46
=
12
= 3:833

S
We can interpret the mean of the data values as the fulcrum or center of gravity in a balance scale as
shown below.

1
DM
P
1 2 3 4 5 6 7 8

mean = 3:8333
PU

Example 33.
Calculate the General Weighted Average (GWA) of
Course Grade Units
Julius Garde for the first semester of school year
BM 112 1.25 3
2019-2020 as shown in the following table.
BM 101 1.00 3
AC 103 1.25 6
Solution. To solve for the GWA, we first consider
MG 101 1.00 3
the entries on the second column of the table as the
EC 111 1.50 3
points xi and the entries in the third column as the
MK 101 1.50 3
corresponding weights wi . By constructing a fourth
FM 111 1.20 3
column consisting of the products wi xi and finding
PE 1 1.00 2
the column totals, we get the table below.

All Rights Reserved. 2020 Abdul, Atienza, et. al.


Lesson 4 56

Course xi wi wi xi
BM 112 1.25 3 3.75
BM 101 1.00 3 3.00
AC 103 1.25 6 7.50
MG 101 1.00 3 3.00
EC 111 1.50 3 4.50
MK 101 1.50 3 4.50
FM 111 1.20 3 3.60
PE 1 1.00 2 2.00

S
Total w = 26 w x = 32:00
P P

We see from the column totals that w = 26 and w x = 32. Therefore, the weighted mean or the
P P

DM
general weighted average (GWA) of Julius Garde for the first semester of AY 2019-2020 is

32
P
wx
x= P = = 1:23:
w 26

Median

Definition 18
The median, usually denoted by x̃, is the middle value of a data set if the observations are
P
arranged either in increasing or decreasing order.

Outliers in the data set do not affect the median. Thus, the median is preferred over the mean as a
PU

measure of central tendency when the data contains outliers. To find the median, begin by listing the
data in order from smallest to largest, or largest to smallest.

If the number of data values, N, is odd, then the median is the middle data value. This value can be
found by rounding N=2 up to the next whole number. If the number of data values is even, there is no
one middle value, so we find the mean of the two middle values (values N=2 and N=2 + 1)

Example 34. Given the scores of 15 students in Mathematics in the Modern World on an exam consisting
of 25 items:
25; 20; 18; 18; 17; 15; 15; 15; 14; 14; 13; 12; 12; 10; 10

Since the data is already arranged in decreasing order and there are 15 observations, hence, we round
15
up = 7:5 to the nearest whole number, which is 8, and take the 8th observation from the left (or
2
right). Therefore, the median is x̃ = 15: In comparison to example 31, the computed mean is 15:2.

All Rights Reserved. 2020 Abdul, Atienza, et. al.


Lesson 4 57

10 11 12 13 14 15 16 17 18 19 20 21 22 23 24 25 26

S
mean

median

Month
January
February
March
April
Hours Lost
55
23
24
37
DM
Remark. In general, the median need not equal the mean.

Example 35. The data given below is the total number of hours lost due to tardiness and absences of
employees in a company in a given year. Find the median.

Solution. If the data are arranged in increasing order, we have

20; 23; 24; 27; 30; 32; 37; 37; 40; 48; 42; 55:

May 37
June 48
Since there are 12 observations (even), we take note of the two
P
July 42 middle observations then compute
August 27
September 20
32 + 37
October 40 x̃ = = 34:5:
November 30 2
PU

December 32

Therefore, the median number of hours lost due to tardiness and absences of employees in a company
in the given year is 34:5 hours.

Mode

Definition 19
The mode is the most frequent observation in a given data set.

Outliers in the data set do not affect the mode. It is possible that the mode of a data set does not
exist, and it is not always unique. It is an appropriate measure of average for data measured only in the
nominal level. We will denote mode using the symbol x̂.

All Rights Reserved. 2020 Abdul, Atienza, et. al.


Lesson 4 58

Example 36. Suppose that we wanted to know the “average color” of cars used by the residents in a
given village. In our vehicle color survey, we collected the following data.
Color Frequency
Blue 3
Green 5
Red 4
White 3
Black 2
Grey 3

S
Since color of vehicles are measured up to the nominal level, the most appropriate measure for the
“average color” is then the mode. The most frequent color is Green, a total of 5 vehicles. Therefore, the
“average color” in our survey data must be Green.

4.9
DM
It is possible for a given data set to have more than one modes. Such a data set is said to be multimodal.
If a given set has only one mode, the data set is unimodal. If it has two modes, the data set is bimodal,
and so on.

Measures of Dispersion or Variability


Measures of dispersion are descriptive summary measures that helps us characterize the data set in terms
of how varied the observations are from the center. If its value is small, then this indicates that the
observations are not too different from the center. On the other hand, if its value is large, then this
indicates that the observations are very different from the center or that they are widely spread out from
P
the center.

Range
PU

Definition 20
The range is the difference between the largest and the smallest observations or items in a set of
data.

The range of a data set is easy to compute, but it is a limited measure because it depends on only two
of the numbers (the highest and the lowest) in the data set. Hence, the range can easily be affected
by outliers. Also, it does not provide any information regarding the concentration of the data from the
center.

Example 37. The following are scores of 20 coming from two different sections, 10 from each section,
in a 50-item exam in MMW.
section 1 40 38 42 40 39 39 43 40 39 40
section 2 46 37 40 33 42 36 40 47 34 45

All Rights Reserved. 2020 Abdul, Atienza, et. al.


Lesson 4 59

For section 1, the highest score is 43, while the lowest score is 38. Thus,

range = 43 − 38 = 5:

On the other hand, for section 2, the highest score is 47, while the lowest score is 33. Thus,

range = 47 − 33 = 14:

Therefore, the scores of students surveyed from section 2 gets a wider range than those of students
surveyed from section 1.

S
Variance and Standard Deviation

Suppose that the center of a population data set {x1 ; x2 ; : : : ; xN } is best described by the arithmetic

would like to compute for

i=1
DM
mean — and that our goal is to get the average “distance” of each data point xi form —. Naturally, we

1 X

(xi − —) =
N

N i=1
(xi − —):

However, using the properties of summations, and the fact that n— = x1 + x2 + · · · + xN we can check
that
N N
X

i=1
xi −
N
X

i=1
— = N— − N— = 0:

In other words, the sum of the deviations from the mean is 0, and therefore, we cannot have a meaningful
measure of variability this way. The reason behind this fact is that some of the deviations from the mean
P
are negative (those which are to the left of the mean) and some are positive (those which are to the right
of the mean) and they cancel each other out. However, we can work our way out of this unfortunate
situation if we can ignore the signs of these deviations. One way to do this is to take the square these
PU

deviations from the mean. We then have the following definition.

Definition 21
The variance of a population data set {x1 ; x2 ; : : : ; xN } with population mean — is defined as

N
1 X
ff 2 = (xi − —)2 :
N i=1

On the other hand, the variance of a sample data set {x1 ; x2 ; : : : ; xn } with sample mean x is
defined as n
2 1 X
s = (xi − —)2 :
n − 1 i=1

As we may have noticed, the formula for the sample variance differs significantly from the formula for

All Rights Reserved. 2020 Abdul, Atienza, et. al.


Lesson 4 60

the population variance mainly because of the divisor n − 1. The reason behind this is rather technical
and mathematical in nature. Simply taken, the divisor n − 1 removes the “bias” in s 2 when we want it
to estimate ff 2 for the purposes of making inferences.

Notice that the variance is a nonnegative quantity because it came from averaging squared quantities.
We also realize that there is one major drawback to using the variance. If we follow the steps in calcu-
lating the variance, we find that the variance is measured in terms of square units because we took the
squares of the deviation. For example, if our sample data is measured in terms of meters, then the units
for a variance would be given in square units.

S
In order to standardize the units, we can take the square root of the variance to eliminate the problem of

Definition 22
DM
squared units, and gives us a measure of the spread that will have the same units as our original sample
or population data.

The population (sample) standard deviation is the nonnegative square root of the the pop-
ulation (sample) variance. In symbols,
√ √
ff = ff 2 and s = s 2:
P
PU

Example 38. Using the sample data sets in example 37, determine which section exhibits a greater
variability in terms of standard deviations.

Solution. Let x denote the scores of students sampled from section 1 and let y denote the scores of
students sampled from section 2. To calculate the standard deviations of each sample, we first take note
that the sample means from each section are

400 400
P P
x y
x= = = 40 and y = = = 40:
n 10 n 10

To calculate the sample standard deviation, we construct the following table.

All Rights Reserved. 2020 Abdul, Atienza, et. al.


Lesson 4 61

x y x −x y −y (x − x)2 (y − y )2
40 46 0 6 0 36
38 37 −2 −3 4 9
42 40 2 0 4 0
40 33 0 −7 0 49
39 42 −1 2 1 4
39 36 −1 −4 1 16
43 40 3 0 9 0
40 47 0 7 0 49
39 34 1 36

S
−1 −6
40 45 0 5 0 25
x = 400 y = 400 (x − x)2 = 20 (y − y )2 = 224
P P P P

s =

DM
Therefore, the sample variance for the sample from section 1 is

2
P
(x − x)2
n−1

while the sample variance for the sample from section 2 is

2
s =
P
(y − y )2
n−1
=
=
20

9
9

224
= 2:2222;

= 24:8888:

Taking square roots, we find that the sample standard deviations of section 1 and section 2 respectively
√ √
are 2:2222 ≈ 1:49 and 24:8888 ≈ 4:99. We can conclude that for these samples, the one from
P
section 1 exhibits the lesser variability than that from section 2. We comment that even though the two
samples have equal means, the standard deviations showed the actual difference between the two data
sets.
PU

All Rights Reserved. 2020 Abdul, Atienza, et. al.


Lesson 4 62

Assessment
1. A research objective is presented. For each,identify the (a)population and (b) sample in the study.

(a) A polling organization contacts 2141 male university graduates who have a white-collar job
and asks whether or not they had received a raise at work during the past 4 months.
(b) A quality-control manager randomly selects 70 bottles of ketchup that were filled on July 17
to assess the calibration of the filling machine.
(c) Every year the PSA releases the Current Population Report based on a survey of 50,000
households. The goal of this report is to learn the demographic characteristics, such as

S
income, of all households within the Philippines.

2. Determine the level of measurement of each variable.

DM
(a) birth order among siblings in a family
(b) favorite movie
(c) volume consumption of water used by a household in a day
(d) eye color
(e) number of siblings

3. Determine the type of sampling used.

(a) A member of Congress wishes to determine her constituents’ opinion regarding estate taxes.
P
She divides her constituency into three income classes: low-income households, middle-income
households, and upper-income households. She then takes a simple random sample of house-
holds from each income class.
PU

(b) A college official divides the student population into five classes: freshman, sophomore, junior,
senior, and graduate student. The official takes a simple random sample from each class and
asks the members opinions regarding student services.
(c) The presider of a guest-lecture series at a university stands outside the auditorium before a
lecture begins and hands every fifth person who arrives, beginning with the third, a speaker
evaluation survey to be completed and returned at the end of the program.
(d) To determine his DSL Internet connection speed, Shawn divides up the day into four parts:
morning, midday, evening, and late night. He then measures his Internet connection speed
at 5 randomly selected times during each part of the day.
(e) 24 Hour Fitness wants to administer a satisfaction survey to its current members. Using its
membership roster, the club randomly selects 40 club members and asks them about their
level of satisfaction with the club.

All Rights Reserved. 2020 Abdul, Atienza, et. al.


Lesson 4 63

4. Patricia categorized her spending for this month into four categories: Rent, Food, Fun, and Other.
The percents she spent in each category are pictured here. If she spent a total of PhP 26,000 this
month, how much did she spend on rent?

S
DM
5. You recorded the time in seconds it took for 8 participants to solve a puzzle. The times were:
15.2, 18.8, 19.3, 19.7, 20.2, 21.8, 22.1, 29.4.

(a) Calculate the mean and the median time it took for the 8 participants to solve a puzzle.
(b) Calculate the range and standard deviation of the time it took for the 8 participants to solve
the puzzle.

6. Make up three data sets with 5 numbers each that have:

(a) the same mean but different standard deviations.


P
(b) the same mean but different medians.
(c) the same median but different means.
PU

All Rights Reserved. 2020 Abdul, Atienza, et. al.


Lesson 5 64

Lesson 5: Financial Mathematics

Learning Outcomes
At the end of the lesson, the students are able to

1. to compute the accumulated or present value of a certain amount of money

2. to compute the accumulated or present value of a series of periodic payments (annuity)

3. to construct an amortization schedule when paying off a loan

S
Managing your money takes much thought and planning. All your daily needs must be met, but at
the same time you should consider your long-term goals such as purchasing a house, saving for college

DM
expenses, and investing for retirement.

A component that is common to all financial transactions is the investment of money at interest. When
a bank lends money to you, it charges rent for the money. When you lend money to a bank (also known
as making a deposit in a savings account), the bank pays rent to you for the money. In either case, the
rent is called ”interest”.

Understanding the cost of borrowing money will help you make informed decisions about your personal
finances.
P
5.1 Simple and Compound Interest
PU

The money a bank or other lender is willing to lend you is called the amount of credit extended or the
principal of the loan. The amount of credit and the interest rate that you may obtain depend on the
assurance you can give the lender that you will be able to repay the loan based on any of the following
(1) your credit reputation, (2) your capacity to earn money or income or (3) the security or collateral
you pledge to be paid to the lender in case you default in your payment.

Banks sometimes grant loans even without security, but they require the signature of one or more other
persons, called cosigners or co-debtors, who guarantee the loan will be repaid. For either of the two types
of loans, the secured loan or the cosigner loan, the borrower (and cosigner, if there is one) must sign an
agreement called a personal note or a bank note. This document states the terms and conditions of
the loan.

The most common ways of borrowing money is through

All Rights Reserved. 2020 Abdul, Atienza, et. al.


Lesson 5 65

• a loan paid in periodic installments,

• a credit card,

• a depository account in banks (banks are the borrowers), and

• a stock or a bond (gov’t and private corporations sell bonds to raise funds).

The concept of simple interest is essential to the understanding of short-term transactions (payment/s
is made within a year). Interest is the money the borrower pays for the use of the lender’s money. One
type of interest is called simple interest.

S
Definition 23: Simple Interest
Simple interest is based on the entire amount of the loan for the total period of the loan. The

DM
formula used to find simple interest follows.

Interest = principal × rate × time


I = Prt

where P is the principal amount, r is the interest rate and the nonnegative value of t (t ≥ 0) is
the term of the investment (number of days, months or years for which the money will be lent).

Since simple interest is usually assumed for short-term transactions, the time
P
# of days between two dates
t=
# of days in a year
is considered to be a fraction of a year and is determined under the following different conventions:
PU

• Exact Interest. Exact number of days between dates is used and the number of days in a year is
taken to be 365 days (366 days for leap years).

• Ordinary Interest. Each month is said to have 30 days and the number of days in a year is taken
to be 360 days (i. e. 12 × 30).

• Banker’s Rule or Merchant’s Rule. Exact number of days between dates is used and the num-
ber of days in a year is taken to be 360 days. Under this rule, an investor or lender mathematically
yields the highest amount of interest so banks would prefer this method.

Example 39. Suppose that Feli borrowed PhP 25,000 from the Provident Fund on March 8 and pays
the entire sum including interest on October 3 of the same year, and that the interest rate is 5%. Find the
amount of interest earned, if it is computed using (a) exact simple interest (b) ordinary simple interest

All Rights Reserved. 2020 Abdul, Atienza, et. al.


Lesson 5 66

and (c) banker’s rule. Assume non leap year.

Solution: From March 8 to October 3, there are exactly N = 23 + 30 + 31 + 30 + 31 + 31 + 30 + 3 = 209


days and under ordinary interest, there are N = 22 + 30 + 30 + 30 + 30 + 30 + 30 + 3 days. Thus the
amount of interest is
209
(a) I = 25; 000 × 0:05 × 365
= PhP 715.75 ;
205
(b) I = 25; 000 × 0:05 × 360
= PhP 711.81 ;
209
(c) I = 25; 000 × 0:05 × 360
= PhP 725.69 .

S
Remarks:

DM
(1) Banks make more money in loans by charging more profitable interest on the principal amount
borrowed than the interest they pay on savings/depository accounts except for time deposits or
certificate of deposits wherein a higher interest is paid. Can you check why?

(2) The amount of interest per period accrued under simple interest is fixed or constant (i.e. it is always
a percentage of the principal amount or mathematically, P × r every period). Note: Review how
to convert interest rates expressed in percent (%) to decimal.

Definition 24: Accumulated and Present Value


In an investment transaction, the value of a principal plus the total interest accrued during the
P
term is termed as its accumulated value or future value and we shall denote it by A. Thus,

A=P +I
PU

where P is the principal amount and I is the interest earned. Suppose at some time in the future,
we would like to obtain a certain amount of money. We will discount interest from the accumulated
value to determine its present value or the amount to be invested to earn the desired interest.
Moreover, the present value of a certian amount at the beginning of the term is the principal value
(P = A − I).

All Rights Reserved. 2020 Abdul, Atienza, et. al.


Lesson 5 67

Theorem 2: Accumulated Value under Simple Interest

If a principal amount P is invested for a term t (t ≥ 0) earns simple interest at a rate r , then its
accumulated value at the end of the term is given by

A = P +I
= P + Prt
A = P (1 + r t):

Investments that involve compound interest may play an important role in reaching some of your long-

S
term financial goals such as investments. An investment is the use of money or capital for income or
profit. We can divide investments into two classes: fixed investments and variable investments. In
a fixed investment, the amount invested as principal may be guaranteed and the interest is computed

DM
at a fixed rate. Guaranteed means that the exact amount invested will be paid back together with any
accumulated interest. Examples of a fixed investment are savings accounts, money market deposit ac-
counts, and certificates of deposit. Another fixed investment is a government savings bond. In a variable
investment, neither the principal nor the interest is guaranteed. Examples of variable investments are
stocks, mutual funds, and commercial bonds.

Definition 25: Compound Interest


Investments made at a compound interest has the property that the interest earned at the end of
one period is automatically invested in the next period to earn additional interest.
P
Let us see how the two differ. Consider an amount of PhP of 1,000 which is invested at a periodic
interest rate of 5%. The amounts in the account after 3 periods assuming (a) simple interest and (b)
compound interest will be
PU

(a) Simple Interest

Year 1 : 1; 000 + 0:05(1; 000) = 1; 050


Year 2 : 1; 000 + 0:05(1; 000) + 0:05(1; 000) = 1; 000(1 + 0:05(2)) = 1; 100
Year 3 : 1; 000 + 0:05(1; 000) + 0:05(1; 000) + 0:05(1; 000) = 1; 000(1 + 0:05(3)) = 1; 150
Indeed, from ??, the accumulated value of this principal amount at time t(t ≥ 0) will be A =
P (1 + r t).

(b) Compound Interest

All Rights Reserved. 2020 Abdul, Atienza, et. al.


Lesson 5 68

Year 1 : 1; 000 + 0:05(1; 000) = 1; 050


Year 2 : 1; 050 + 0:05(1; 050)) = 1; 050(1 + 0:05) = 1; 102:50
Year 3 : 1; 102:50 + 0:05(1; 102:50) = 1; 102:50(1 + 0:05) = 1; 157:625 ≈ 1; 157:63
Eventually at some time t, we will yield an accumulated value of

A = 1; 000 (1:05)(1:05) · · · (1:05) = 1; 000(1:05)t :


| {z }
t times

Remarks:
• Note that since interest may be compounded more often than once in a year, then the effective

S
interest rate per compounding period is given by nr where r is the compound interest rate and n is
the the frequency of conversion.

DM
• If n = 1, then r is just the effective annual interest rate or simply the effective rate.

• If n = 2, then we say that the interest is compounded semiannually. If n = 4, then it is compounded


quarterly. And if n = 12, then we say that the interest is compounded monthly.

Theorem 3: Accumulated Value under compound Interest

If a principal amount P is invested for a term t (t ≥ 0) earns interest at an effective rate r


compounded n times in a year, then its accumulated value at the end of the term is given by

r
„ «„
r
« „
r
«
A = P 1+ 1+ ··· 1 +
| n n
{z n}
P
n times in t periods
«nt
r

= P 1+ :
n

Thus, the present value of such an amount A is


PU

«−nt
r

P =A 1+
n

Most debts and investments earn compound in- A


compound interest
terest, where interest is earned on both the original
principal and the interest up to that point (or we 1+r • simple interest

say, "interest is compounded"). An investment with 1


compound interest will be worth more every year and
thus will earn more interest. Hence, banks charge
compound interest for long-term transactions.
1 t

All Rights Reserved. 2020 Abdul, Atienza, et. al.


Lesson 5 69

Example 40. Regine invested PhP 80,000 in a savings account with an interest of 1.8% convertible
(compounded) monthly. If she, does not make any deposit nor withdrawal into this account, determine
the amount in the account after 2 years.

Solution. By the previous theorem,


«12×2
0:018

A = 80; 000 1 + ≈ PhP 82,930.23 :
12

S
Example 41. Cyrus would like to invest enough money in a certificate of deposit (CD) now to pay for
his son?s college expenses. If he estimates that he will need PhP 1,500,000 in 6 years, how much should
he invest now in a CD that has a rate of 2.5% compounded quarterly?

Solution. By the previous theorem,

DM „
P = 1; 500; 000 1 +
0:025
4
«4×6
≈ PhP 129,166.48 :

Caution: Convert percentages to decimal carefully. Round off amounts of money to the nearest centavo.
Do not round off in the middle of the computation beacuse the final answer may be off by a huge amount
especially when dealing with large amounts of investments. Have a good scientific calculator available
to aid your computations.
P
5.2 Ordinary Annuities
PU

Some payments are made regularly in more than one periods (monthly, quarterly or annually) as in the
case paying off a car loan.
Definition 26: Annuity
An annuity is an account into which, or out of which, a sequence of scheduled payments is made.

There are many different types of annuities. An annuity may be an investment account that you have
with a bank, insurance company, or financial management firm. Annuities may contain investments in
stocks, bonds, mutual funds, money market accounts, and other types of investments. Annuities are
often used to save for long-term goals such as saving money for college or for retirement. An annuity can
also be used to provide long-term regular payments to individuals. Some lottery jackpots and professional
athletes’ salaries are paid out over time from annuities. Retirees may invest some of their retirement
savings (pension) into an annuity and then receive monthly payments that come from that annuity rather
than withdraw everything as one lump sum.

All Rights Reserved. 2020 Abdul, Atienza, et. al.


Lesson 5 70

Consider an annuity with regular payouts of R at the end of each of the n periods such that it earns
interest at an effective rate of r per period. Then its present and accumulated values are obtained as
follows

P = R(1 + r )−1 + R(1 + r )−2 + · · · + R(1 + r )−n


(this is a geometric series with n terms and a common ratio (1 + r )−1 )
1 − (1 + r )−n
= R×
1 − (1 + r )−1
1 − (1 + r )−n

S
P = R×
r

and

= R×
1 − (1 + r )
(1 + r )n − 1
(1 + r ) − 1
(1 + r )n − 1
DM
A = R + R(1 + r ) + +R(1 + r )2 · · · + R(1 + r )n−1
(this is a geometric series with n terms and a common ratio (1 + r ))

= R×
1 − (1 + r )n

A = R×
r
P
Definition 27: Ordinary Annuity
An annuity into which equal payments are made at regular intervals, with the interest compounded
at the end of each interval and with a fixed interest rate for each compounding period, is called
PU

an ordinary annuity or a fixed annuity.

Theorem 4: AV and PV of an Ordinary Annuity


If an ordinary annuity has regular payments of R at the end of each of the n periods and earns
interest at an effective rate of r per period. Then its present and accumulated values are given by

(1 + r )n − 1 1 − (1 + r )−n
A=R× and P =R× :
r r

Remarks:

• Since annuities are mostly long-term transactions, we will assume that interest is compounded at
an effective rate of r every period

All Rights Reserved. 2020 Abdul, Atienza, et. al.


Lesson 5 71

• The n such regular payments noted above are said to be made at the end of each payment period
such that no additional payments nor withdrawals are made during the term.

• All payments in the ordinary annuities considered in this material are considered guaranteed pay-
ments. Such annuities are referred to as certain annuities. If payments depend on the occurrence
of an event then we call it a contingent annuity (e.g. payments in an annuity from a pension
fund are paid out as long as the annuitant is alive). The latter is not covered in this material and
may be discussed in details in an advanced course in financial mathematics as it will require deeper
knowledge of probabilities.

S
Example 42. JB and Maryvhic are depositing PhP 10,000 each quarter in an ordinary annuity that pays
4% interest compounded quarterly. Determine the accumulated amount in this annuity after 5 years.
How much total interest was earned?

DM
Solution. Note that there are n = 4 × 5 = 20 quarterly payments and the quarterly rate is 1% so the
value of the annuity after 5 years is

A = 10; 000 ×

The total interest is hence equal to


(1:01)20 − 1
0:01
= PhP 220,190.04 :

I = Accumulated Amount − Total Payments


P
= 220; 190:04 − 10; 000(20)
= 220; 190:04 − 200; 000
PU

I = PhP 20,190.04 :

Example 43. Jeremiah is willing to pay a loan with PhP 3,000 monthly installments for 3 years at a
rate of 8% convertible monthly. How much money can he borrow today?

Solution. Note that there are n = 12 × 3 = 36 monthly payments and the monthly rate is 8%/12 so the
value of the loan is
0:08 36
„ «
1− 1+
P = 3; 000 × 12 ≈= PhP 95,735.42 :
0:08
12

All Rights Reserved. 2020 Abdul, Atienza, et. al.


Lesson 5 72

5.3 Loan Repayment or Paying Off a Debt


A loan is an arrangement in which a lender gives money or property to a borrower, and the borrower
agrees to return the property or repay the money, usually along with interest, at some future point(s) in
time. Various methods of repaying a loan are possible. We will consider two of them: The amortization
method and the sinking fund method.

Definition 28: Amortization Method


In the amortization method the borrower makes installment payments to the lender. Usually
these payments are at a regularly spaced periodic intervals; the progressive reduction of the amount

S
owed is described as the amortization of the loan. Examples include car loan, home mortgage
repayment.

DM
When using the amortization method, the payments form an annuity whose present value is equal to the
original amount of the loan. In this section, we want to determine the unpaid balance, also referred to
as the outstanding loan balance or unpaid principal at any time after the inception of the loan.
There are two approaches used in finding the amount of the outstanding balance: the prospective and
the retrospective method which in general, are equivalent.
(1) Pospective Method: the outstanding loan balance at any point in time is equal to the present
value at that date of the remaining payments.

(2) Retrospective Method: the outstanding loan balance at any point in time is equal to the
original amount of the loan accumulated to that date less the accumulated value at that date of
P
all payments previously made.
Theorem 5: Formula for Computing for the Outstanding Loan Balance
Let L be the principal amount borrowed which is to be repaid in n periods at an effective interest
PU

rate of r per period. The prospective outstanding balance at the end of the kth period is

1 − (1 + r )−(n−k)
Bk = R ×
r

which is the present value at time k of the remaining n − k "future" payments.

The prospective outstanding balance at the end of the kth period is

(1 + r )k − 1
Bk = L(1 + r )k − R× :
{z r
| {z }
| }
value of the loan accumulated value of all
at time k "past" payments at time k

All Rights Reserved. 2020 Abdul, Atienza, et. al.


Lesson 5 73

Example 44. A loan is being repaid with level monthly payments of PhP 1,000. Calculate the out-
standing balance of the loan if there are 12 payments left. The next payment will be paid one year from
now and the interest rate is 6% convertible monthly.
Solution. There are 12 monthly payments and we have no information about how much the principal
loan balance is and how many payments have been made previously so we compute for the outstanding
loan balance as follows
«−12
0:06

1− 1+
B = 1; 000 × 12 ≈ PhP 11,618.93 :
0:06
12

S
DM
Example 45. A loan of 200,000 is being repaid with quarterly payments of 20,000 at the end of each
year. The interest rate charged on the loan is 8% compounded quarterly. Calculate the outstanding
balance of the loan immediately after the 5th payment.

Solution. There were already 5 quarterly payments done and we have no information about how many
more payments are remaining so we compute for the outstanding loan balance retrospectively as follows
«5
0:08

P

0:08
«5 1+ −1
B = 200; 000 1 + − 20; 000 × 4 ≈ PhP 116,735.36 :
4 0:08
4
PU

When a loan is being repaid with the amortization method, each payment is partially a repayment of
principal and partially a payment of interest. Determining the amount of each for a payment can be im-
portant (for tax purposes, for example). An amortization schedule is a table which shows the division of
each payment into principal and interest, together with the outstanding loan balance after each payment
is made.

Let L be the principal loan balance (i.e. the outstanding loan balance at time 0) being repaid with
regular installment payments of R, then an amortization schedule would look the table below.

All Rights Reserved. 2020 Abdul, Atienza, et. al.


Lesson 5 74

Payment Interest Principal Outstanding


Period Amount Paid Repaid Balance
0 B0 = L
1 R I1 P1 B1
2 R I2 P2 B2
.. .. .. .. ..
. . . . .
n R In Pn Bn = 0

1 − (1 + r )−n 1 − (1 + r )−n
!
Total nR nR − R × R×

S
r r

Remarks:

DM
(1) The amortization method is the most common method of loan repayment. The fundamental
principle behind it is simple. When a payment is made, it must be first applied to pay
interest due and then any remaining part of the payment is applied to pay principal.

(2) Fill up the table row by row from the top using the following:

• The regular installment payment is given by

R=
L
1 − (1 + r )−n
!

r
P
where r is the effective rate of interest per period and n is the total number of payment
periods.
PU

• On row/period t, the payment is a sum of interest paid and principal repaid (i.e. R = It +Pt )
• At period t, the interest paid is the interest rate effective per period r times the previous
balance or
It = r × Bt−1 ;

the principal repaid is


Pt = R − It ;

while the outstanding balance is the former outstanding balance less the current principal
repaid (i.e. Bt = Bt−1 − Pt ).

(3) The total principal repaid is the loan amount. The total interest paid is the difference between the
total payments made less the loan.

(4) The balance is fully paid off or is zeroed out by the end the loan’s term.

All Rights Reserved. 2020 Abdul, Atienza, et. al.


Lesson 5 75

Example 46. Consider a loan PhP 30,000 with level payments to be made at the end of every 2 months
for 10 months at a bimonthly rate of 5% (need not divide by 6, this is already the effective rate every
2-month period unless we say "compounded bimonthly"). Construct a loan amortization schedule for this.

Solution. We first compute for the amount of each level payment. This is obtained as follows

30; 000
R= ! ≈ PhP 6,929.24 :
1 − (1 + 0:05)−5
0:05

Immediately fill the payment column with this amount.

S
Payment Interest Principal Outstanding
Period Amount Paid Repaid Balance

DM
0 PhP 30,000.00
1 PhP 6,929.24 PhP 1,500.00 PhP 5,429.24 PhP 24,570.76
2 PhP 6,929.24 PhP 1,228.54 PhP 5,700.70 PhP 18,870.06
3 PhP 6,929.24 PhP 943.50 PhP 5,985.74 PhP 12,884.32
4 PhP 6,929.24 PhP 644.22 PhP 6,285.02 PhP 6,599.30
5 PhP 6,929.24 PhP 329.96 PhP 6,599.28 PhP 0.00

Remarks: Observe the following from the previous example.

• The last principal repaid is off by PhP 0.02 as an effect of rounding off the level payment computed
P
to the nearest centavo. As a rule of thumb, you may manually adjust this by adding this to the
last principal repaid. In practice, tables are created using spreadsheets or computer programs to
avoid this rounding off errors.
PU

• The final balance is 0. The level payment fully pays off the loan as intended.

• As the balance decreases over time the amount of interest due in each period decreases.

• As the interest due goes down, the amount of principal paid in each period increases.

• This method is typically applied to loans with level payments but the payments also may change
or vary over time as may apply to a payor who wish to make payment adjustments in the middle
of the term.

Example 47. Consider the loan PhP 30,000 in the previous example. Suppose the debtor can only pay
PhP 5,000 for each of the first 2 periods and catch up with a higher payment for the final three periods.
Construct a loan amortization schedule for this.

Solution. We first fill the first two payments with PhP 5,000.

All Rights Reserved. 2020 Abdul, Atienza, et. al.


Lesson 5 76

Payment Interest Principal Outstanding


Period Amount Paid Repaid Balance
0 PhP 30,000.00
1 PhP 5,000.00 PhP 1,500.00 PhP 3,500.00 PhP 26,500.00
2 PhP 5,000.00 PhP 1,325.00 PhP 3,675.00 PhP 22,825.00
3
Thus, the outstanding balance after two payments is PhP 22,825.00 which will be repaid with higher
revised payments determined as follows

22; 825

S
R= ! ≈ PhP 8,381.54 :
1 − (1 + 0:05)−3
0:05
As expected, it is higher than the first two payments.

Period
0
1
2
3
4
Payment
Amount

DM
PhP 5,000.00
PhP 5,000.00
PhP 8,381.54
PhP 8,381.54
Interest
Paid

PhP 1,500.00
PhP 1,325.00
PhP 1,141.25
PhP 779.24
Principal
Repaid

PhP 3,500.00
PhP 3,675.00
PhP 7,240.29
PhP 7,602.30
Outstanding
Balance
PhP 30,000.00
PhP 26,500.00
PhP 22,825.00
PhP 15,584.71
PhP 7,982.41
5 PhP 8,381.54 PhP 399.12 PhP 7,982.41 PhP 0.00
P
Note that we adjusted the last principal repaid manually by PhP 0.01 so the final balance will be 0.

Definition 29: Sinking Fund


An alternative for repaying a loan in installments by the amortization method, a borrower can
PU

accumulate a fund which will exactly repay the loan in one lump sum at the end of a specified
period of time. This fund is called a sinking fund. It is generally required that the borrower
periodically pay interest on the loan, sometimes referred to as a service.

In this section, we shall only consider a sinking fund with fixed regular contributions. Because the balance
in the sinking fund could be applied against the loan at any point, the net amount of the loan is equal
to the original amount of the loan minus the sinking fund balance. It is known that if the rate of interest
paid on the loan equals the rate of interest earned on the sinking fund, then the amortization method
and the sinking fund method are equivalent.

Let L be the loan amount payable via sinking fund deposits D earning interest at an effective rate k and
interest due to the lender at an effective rate r per period. A sinking fund schedule should look like the
one below.

All Rights Reserved. 2020 Abdul, Atienza, et. al.


Lesson 5 77

Interest Sinking Fund Interest Amount Net Amount


Period Payment Deposit on the SF in the SF of the loan
0 SB1 = 0 B0 = L
1 Lr D I1 SB1 B1
2 Lr D I2 SB2 B2
.. .. .. .. .. ..
. . . . . .
n Lr D In SBn = L Bn = 0

Remarks:

S
(1 + k)n − 1
(1) If L is the loan amount then L = D × where n is the term of the loan and k is the
k
effective rate of interest per payment period on the sinking fund.

DM
(2) Fill up the table row by row from top using the following:

• The interest payable to the lender each period is L × r .

• The sinking fund deposit is given by

D=
L
(1 + k)n − 1
k
!:

• The interest earned on the sinking fund balance during period t is It = k × SBt−1 where k
P
is charged to the previous sinking fund balance.

• At time t, the balance on the sinking fund is


PU

SBt = SBt−1 + D + It :

• At time t the net amount of the loan or outstanding balance is the loan amount less the
current amount saved in the sinking fund or

Bt = L − SBt :

Example 48. Consider a loan PhP 30,000 to be paid with level interest due to the lender at the end
of every 2 months for 10 months at a bimonthly rate of 5% (need not divide by 6, this is already the
effective rate every 2-month period unless we say "compounded bimonthly") along with deposits to a
sinking fund that earns a bimonthly rate of 4%. Construct a loan sinking fund schedule for this.

Solution. The interest due to the lender each period is L × r = 30; 000 × 0:05 = PhP 1; 500. We then

All Rights Reserved. 2020 Abdul, Atienza, et. al.


Lesson 5 78

compute for the amount of each level sinking fund deposit. This is obtained as follows

30; 000
D= ! ≈ PhP 5,538.81 :
(1 + 0:04)5 − 1
0:04

Immediately fill the interest payment and sinking fund deposit columns with these corresponding amounts
and complete the remainder of the table.

Interest Sinking Fund Interest Amount Net Amount


Period Payment Deposit on the SF in the SF of the loan

S
0 PhP 0.00 PhP 30,000.00
1 PhP 1,500.00 PhP 5,538.81 PhP 0.00 PhP 5,538.81 PhP 24,461.19
2 PhP 1,500.00 PhP 5,538.81 PhP 221.55 PhP 11,299.17 PhP 18,700.83

5.4
(1) Stocks
3
4
5
PhP 1,500.00
PhP 1,500.00
PhP 1,500.00

DM
PhP 5,538.81
PhP 5,538.81
PhP 5,538.81

Other Applications of Financial Mathematics


PhP 451.97
PhP 691.60
PhP 940.83
PhP 17,289.95
PhP 23,520.36
PhP 30,000.00
PhP 12,710.05
PhP 6,479.64
PhP 0.00

When the owners of a company wish to raise money to expand their company, they often decide
P
to sell part of the company to investors. When an investor purchases a portion of a company, the
investor is said to own stock in the company. The unit of measure of the stock is called a share. By
buying shares of stock, an investor is becoming a part owner, or shareholder, of the company.
PU

Large companies may have many millions or even billions of shares of stock available for trading to
the general public. For example, as of 2020, there are about 4.334 billion shares of Apple, Incor-
porated, owned by thousands of investors. On July 30, 2020, each of these shares was worth $ 380.16.

Investing in stocks over a long period of time is usually a good investment. However, since the price
of stocks may go down as well as up, investing in stocks involves some risk of losing some or all of
your investment.

(2) Bonds
A bond is a type of loan. When government agencies or corporations need money, they often borrow
money from investors by selling, or issuing, bonds. When an investor purchases a bond, the investor
is actually lending money to the bond’s issuer. The issuer agrees to pay the investor a certain interest

All Rights Reserved. 2020 Abdul, Atienza, et. al.


Lesson 5 79

rate over a stated period of time, usually from 1 to 30 years. The date on which the issuer repays
the loan is called the date of maturity.

Although bonds are generally considered safer investments than stocks, they do have some risks.
On rare occasions, issuers may fail to make interest payments or may fail to return the investment
entirely. A more common risk is that the value of a bond may decrease if interest rates increase.
Such a decrease may cause investors to lose some of their investment if they decide to sell their bond
before the date of maturity. In general, though, bonds offer a very stable investment that usually
provide a higher return on investment than savings accounts or certificates of deposit without many

S
of the risks associated with investing in stocks.

(3) Mutual Funds

DM
A mutual fund is an investment tool that enables investors to indirectly own a wide variety of
stocks, bonds, or other investments. When investors purchase shares in a mutual fund, they are
actually placing their money in a pool along with many other investors. The investments within
a mutual fund are called the mutual fund’s portfolio. The investors of a mutual fund share the
gains and losses from the investments within the portfolio. There are some distinct advantages to
investing in mutual funds rather than invest- ing in individual stocks and bonds. First, investors in
mutual funds have their money managed by full-time professionals. Second, because large sums of
money are managed within a mutual fund, costs related to investing, known as commissions, are
generally lower than they are for purchasing individual stocks and bonds. Third, when investors
purchase shares in a mutual fund, they are indirectly purchasing shares in a multitude of stocks or
P
bonds. This diversification can greatly help to reduce some of the risks of investing.
PU

One disadvantage of mutual fund investing is the potential to miss out on a large return on invest-
ment. In general, though, investing in mutual funds is considered an excellent way to begin investing
and to maintain diverse ownership in a variety of investments.

(4) Home Equity

Most of us are aware that one big dream is to own your own home. What we may not realize is that
in addition to having a place to call your own, several important financial benefits occur when you
own your own home.

• First, instead of paying rent to someone else, you make a mortgage payment that builds the
equity in your home. Equity is the difference between the appraised value of your home and
your loan balance, and it usually increases with each payment you make. As years go by, this
equity may also help you qualify for other loans such as college and car loans.

All Rights Reserved. 2020 Abdul, Atienza, et. al.


Lesson 5 80

• Second, the interest and real estate taxes you pay (in most cases) are deductible on your income
tax returns. These deductions can add up to significant savings each year and may result in a
larger tax refund.
• Finally, over time you can typically expect your home to increase in value. Thus,your home not
only becomes your place of dwelling; in most cases, it also serves as a wise financial investment.

(5) Credit Scores


When consumers borrow money by obtaining car loans, home mortgages, or other loans or through
the use of credit cards, the lender takes a risk by giving money to the borrower in exchange for the

S
borrower’s promise to repay the loan plus interest. To minimize their risk, lenders do research on
borrowers to determine the likelihood the borrowers will default, or fail to repay the loans.

DM
Banks submit our basic credit data to the Credit Information Corporation (CIC), an Office created
under Republic Act (R.A.) No. 9510, also known as the Credit Information System Act. CIC assign
a credit score to an individual after researching the individual?s credit history, income, age, and other
factors. Lenders use the information from CIC to determine a borrower’s credit worthiness. A high
credit score suggests that the individual is more likely to repay a loan and often enables the borrower
to get lower interest rates on loans.
P
PU

All Rights Reserved. 2020 Abdul, Atienza, et. al.


Lesson 5 81

Assessment
1. Find the missing value.

Principal Rate Time Simple Interest


(a) 15% 7 years P,65,625
(b) P500 2.5 years P40
(c) P1,250 5% P375
(d) P900 9 12 % 18 months

S
2. (Simple Interest) Consider a 2,200 pesos at 7% interest for 100 days. Find the simple interest
using:

(a) Exact Interest

(b) Ordinary Interest

(c) Banker’s Rule DM


3. Suppose that you have a choice of two loans: one at 5% simple interest for 6 years, and one at
6% simple interest for five years. Which will result in the smaller future value? Does it depend on
the principal?

4. (Compound Interest) Find the compound interest and future value for each.
P
Principal Rate Compounded Time
PU

(a) P825 4% Annually 10 years


(b) P3,250 2% Semiannually 5 years
(c) P625 8% Quarterly 12 years
(d) P750 3% Monthly 1 year

5. Find the future value of an annuity if you invest P 500 quarterly for 20 years at 5% interest
compounded quarterly.

6. (Amortization) A store was purchased for P 725; 000 and the buyer made a 10% down payment.
The balance was financed with a 6:35% loan for 27 years. Find the monthly payment.

7. You decide to buy P1,800,00 home. If you make a 25% down payment, you can get a 20-year
mortgage at 9%, but if you can make a 10% down payment, you can get a 25-year mortgage at
7%. Which is the better options for you?

All Rights Reserved. 2020 Abdul, Atienza, et. al.


Lesson 5 82

8. Andrew borrowed PhP 25,000.00 and agrees to repay the loan by level monthly installments for
10 months at 8.4% interest convertible monthly. Construct a loan amortization schedule for this
loan.

Computation for the Level Payment R


t Payment Interest Principal Balance
0
1
2
3

S
4
5
6
7
8
9
10 DM
P
PU

All Rights Reserved. 2020 Abdul, Atienza, et. al.


Lesson 6 83

Lesson 6: Voting Methods and Apportionment

Learning Outcomes
At the end of the lesson, the students are able to

1. interpret information from a preference table

2. state the fairness criteria

3. determine the winner of an election using some voting methods

S
4. decide if an election violates any of the fairness criteria

5. apportion seats using some apportionment methods and state apportionment flaws

6.1 Voting Methods

DM
In many decision making situations we encounter, it is necessary to gather the preference of the group.
Voting seems like such s simple idea. If you have two options, whichever gets the most votes wins. But
like most things in the world, election turn out to be not as simple as it appear. While the basic idea
of voting is fairly universal, the method by which those votes are used to determine a winner can vary,
especially when there are more than two choices. In deciding upon a winner, there is always one main
goal: to reflect the preferences of the people in the most fair way possible. The question is, “Is there
P
any voting method that satisfies all the fairness criteria?”

Preference Ballot and Preference Table


PU

Social choice theory looks at processes by which different and conflicting choices of members of a
group are consolidated into a single choice of the group.

A preference ballot is a ballot on which each voter ranks all eligible candidates, from first to last place,
with no tied.

Example 49.
Rank Vote
1st B
2nd D
3rd A
4th C
A preference table is a table showing how many times each possible ballot was submitted.

All Rights Reserved. 2020 Abdul, Atienza, et. al.


Lesson 6 84

Example 50.

Number of Votes
Rank 6 5 3 1
1st A B D C
2nd B D B A
3rd C A A B
4th D C C D

The table shows how 15 voters (6+5+3+1) cast their votes. Six voters ranked A first, B second, C
third, D fourth, five voters ranked B first, D second, A third, C last, and so on.

S
A voting method is a mathematical procedure that uses data from the preference table to determine
a winner.

tonicity, irrelevant alternatives).

The Plurality Method


DM
For the following discussion of each voting method (plurality, Borda count, plurality-with-elimination,
pairwise comparison), a fairness criterion will also be included (head-to-head comparison, majority, mono-

The simplest method of determining a winner in an election with three or more candidates is called the
plurality method.
P
The Plurality Method
In an election with three or more candidates that uses the plurality method to determine a
winner, the candidate with the most first-place votes is the winner.
PU

Example 51. Suppose we have four candidates (A, B, C, and D) and the following preference table.

Number of Votes
Rank 14 10 8 4 1
1st A C D B C
2nd B B C D D
3rd C D B C B
4th D A A A A

A wins using plurality.

Remarks.

All Rights Reserved. 2020 Abdul, Atienza, et. al.


Lesson 6 85

1. Plurality means "more votes than any other candidate" while majority means "more than 50% of
the votes".

2. Every majority is plurality.

3. Not every plurality is a majority.

Head-to-Head Comparison Criterion


The head-to-head comparison criterion states that if a particular candidate wins all head-to-

S
head comparisons with all other candidates, then that candidate should win the election.

Example 52. Suppose we have three candidates (A, B, and C) and the following preference table.

By plurality method, B is the winner.


DM Rank
1st
2nd
3rd
Number of Votes
4
B
C
A
7
A
C
B
5
C
A
B
4
B
A
C
P
Does this election violate the head-to-head comparison criterion?

The idea is to compare all combinations of two candidates at a time to see which is preferred in a
head-to-head matchup without the third candidate involved.
PU

First, compare A and B. Twelve voters listed A higher than B while 8 voters listed B higher than A. So,
candidate A win a head-to-head matchup with candidate B.

Next, compare A and C. There are 11 voters who listed A higher than C and 9 who listed C higher than
A, so A also wins a head-to-head matchup with C.

Without comparing B and C, we can see that the head-to-head comparison criterion is violated: A defeats
both B and C but candidate A did not win the election using the plurality method.

Remark. The head-to-head comparison criterion doesn’t say that the winner of an election has to defeat
every opponent head-to-head. It says that if there is a candidate that does defeat all others head-to-head,
that candidate should win the election.

All Rights Reserved. 2020 Abdul, Atienza, et. al.


Lesson 6 86

The Borda Count Method

Another method of voting when there are three or more alternatives is called the Borda count method.
This method was developed by a French naval captain and mathematician, Jean-Charles de Borda. How-
ever, the method was used to elect Roman senators about 1600 years before Borda was born.

The Borda Count Method


The Borda count method of voting requires the voter to rank each candidate from most favorable
to least favorable then assigns 1 point to the last-place candidate, 2 points to the next-to-the-last
candidate, 3 points to the third-from-the-last candidate, etc. The points for each candidate are

S
totaled separately, and the candidate with the most points wins the election.

DM
Example 53. Suppose we have four candidates (A, B, C, and D) and the following preference table.

Rank
1st
2nd
3rd
4th
A
B
C
D
Number of Votes
14 10
C
B
D
A
8
D
C
B
A
4
B
D
C
A
C
D
B
A
1

Use the Borda count method to determine the winner.


P
Solution. Since there are four candidates, we assign 1 point for the fourth choice, 2 points for the
third choice, 3 points for the second choice and 4 points for the first choice. Multiply the number of
PU

votes by the number of choice for each candidate to get the total points. Hence we have the Borda scores,

Number of Votes
Rank 14 10 8 4 1
1st A:56 C:40 D:32 B:16 C:4
2nd B:42 B:30 C:24 D:12 D:3
3rd C:28 D:20 B:16 C:8 B:2
4th D:14 A:10 A:8 A:4 A:1

A gets 79 points (56+10+8+4+1), B gets 106 points (42+30+16+16+2), C gets 104 points (28+40+24+8+4)
and D gets 81 points (14+20+32+12+3).

The winner is B.

All Rights Reserved. 2020 Abdul, Atienza, et. al.


Lesson 6 87

Remark. Make sure to assign the most points to the candidate listed first. It is just very common to
mistakenly assign one point for a first place vote, two points for the second, and so on.
Majority Criterion
The majority criterion states that if a candidate receives a majority of first-place votes then that
candidate should be the winner of the election.

Example 54. Suppose we have three candidates (A, B, and C) and the following preference table.

S
Number of Votes
Rank 11 7 6 4

DM 1st
2nd
3rd
C
A
B
A
B
C
A
C
B
C
B
A

By Borda count method, we have the following total points for each candidate A: 65; B: 39; C: 64
(Verify.) Hence, A is the winner. However, of the 28 ballots cast, 15 listed C first which means that a
majority of voters listed C first. Since C did not win, the majority criterion is violated.

The Plurality-with-Elimination Method


P
The plurality-with-elimination method was designed specifically with the majority criterion in mind;
if no candidate gets a majority of first-place votes, a series of rounds is used in which candidates are
eliminated and votes are recounted. This method is commonly called the instant runoff voting and
PU

some referee to this method as survival of the fittest method.

The Plurality-with-Elimination Method


In the plurality-with-elimination method, the candidate with the majority of first-place votes is
declared the winner. If no candidate has a majority of first-place votes, the candidate(s) with the
least number of first-place votes is eliminated, then the candidates who were below the eliminated
candidate move up on the ballot, and the number of first-place votes is counted again. If a
candidate receives a majority of first-place votes, that candidate is declared the winner. If no
candidate receives the majority of first-place votes, the one with the least number of first-place
votes is eliminated, and the process continues.

Example 55. Suppose we have four candidates (A, B, C, and D) and the following preference table.

All Rights Reserved. 2020 Abdul, Atienza, et. al.


Lesson 6 88

Number of Votes
Rank 14 10 8 4 1
1st A C D B C
2nd B B C D D
3rd C D B C B
4th D A A A A

Use the plurality-with-elimination method to determine the winner.

S
Solution.
Round 1: A has 14 first places, B has 4 first places, C has 11 first places, and D has 8 first places.
Eliminate B.

Rank
1st
2nd
3rd
4th
A
B
C
D
Number of Votes
14 10
C
B
D
A
8
D
C
B
A
4
B
D
C
A
DM
C
D
B
A
1 Rank
1st
2nd
3rd
4th
A

C
D
Number of Votes
14

Round 2: A has 14 first places, C has 11 first places, and D has 12 first places.
10
C

D
A
8
D
C

A
4

D
C
A
C
D

A
1

Eliminate C.
P
Number of Votes Number of Votes
Rank 14 10 8 4 1 Rank 14 10 8 4 1
1st A C D D C 1st A D D
PU

2nd C D C C D 2nd D D
3rd D A A A A 3rd D A A A A

Round 3: A has 14 first places and D has 23 first places.

Number of Votes
Rank 14 10 8 4 1
1st A D D D D
2nd D A A A A

Hence, D wins.

All Rights Reserved. 2020 Abdul, Atienza, et. al.


Lesson 6 89

The Monotonicity Criterion


The monotonicity criterion states that if a candidate wins an election, and a reelection is held in
which the only changes in voting favor the original winning candidate, then that candidate should
still win the reelection.

Example 56. Suppose we have three candidates (X, Y, and Z) and the following preference table.

Number of Votes
Rank 7 13 11 10

S
1st X Z Y X
2nd Z X Z Y
3rd Y Y X Z

DM
Using the plurality-with-elimination method, candidate Y is eliminated in round 1.

With Y eliminated, the preference table will now be

Hence, Z wins with 24 first-place votes.


Rank
1st
2nd
Number of Votes
7
X
Z
13
Z
X
11
Z
X
10
X
Z
P
Now, suppose the first election was declared invalid for some reason, and on a second election, the voters
in column 1 change their ballots in favor of candidate Z and vote ZXY.
The new preference table will now be
PU

Number of Votes
Rank 7 13 11 10
1st Z Z Y X
2nd X X Z Y
3rd Y Y X Z
Here, X is eliminated on the first round.

With X eliminated, the preference table looks like this


Number of Votes
Rank 7 13 11 10
1st Z Z Y Y
2nd Y Y Z Z

All Rights Reserved. 2020 Abdul, Atienza, et. al.


Lesson 6 90

Now, Y is the winner with 21 votes compared to 20 votes for Z.


Observe that in tis example, the plurality-with-elimination method fails the monotonicity criterion. Even
though candidate Z received 7 more votes for the first place , still he lost the election.

The Pairwise Comparison Method

The pairwise comparison method uses a preference table to compare each pair of candidates. We can
use the formula for combination of n distinct objects choose r at a time,n Cr , to determine the number
of pairwise comparisons needed.
The Pairwise Comparison Method

S
The pairwise comparison method of voting requires that all candidates be ranked by the voters.
Then each candidate is paired with every other candidate in a one-to-one contest. For each one-
to-one comparison, the candidate who wins on more ballots gets 1 point. In case of a tie, each

DM
candidate gets 0:5 point. After all possible two-candidate comparisons are made, the points for
each candidate are tallied, and the candidate with the most points wins the election.

Example 57. Suppose we have four candidates (A, B, C, and D) and the following preference table.

Rank
Number of Votes
14 10 8 4 1
1st A C D B C
2nd B B C D D
P
3rd C D B C B
4th D A A A A
Use the pairwise comparison method to determine the winner.
PU

Solution. Since there are four candidates, we will need 4 C2 = 6 pairwise comparisons: A vs B, A vs C,
A vs D, B vs C, B vs D, and C vs D.

First, let us consider A vs B.


A is preferred by 14 over B whereas B is preferred by 23 over A. B wins. Assign 1 point to B.

Next, compare A to C.
A is preferred by 14 over C whereas C is preferred by 23 over A. C wins. Assign 1 point to C.

A vs D.
A is preferred by 14 over D whereas D is preferred by 23 over A. D wins. Assign 1 point to D.

All Rights Reserved. 2020 Abdul, Atienza, et. al.


Lesson 6 91

B vs C
B is preferred by 18 over C whereas C is preferred by 19 over B. C wins. Assign another 1 point for C.

B vs D
B is preferred by 28 over D whereas D is preferred by 9 over B. B wins. Assign another 1 point for B.

Finally, compare C to D
C is preferred by 25 over D whereas D is preferred by 14 over C. C wins. Another 1 point will be assigned
to C.

S
Summarizing the results, we have

Total

DM
Candidate A
Candidate B
Candidate C
Candidate D

Candidate C has the most points, hence, C is the winner.

Irrelevant Alternatives Criterion


1
1
1
1
1 1
2
3
1

The irrelevant alternatives criterion requires that if a certain candidate X wins an election and
one of the other candidates is removed from the ballot and the ballots are recounted, candidate
P
X still wins the election.

Example 58. Determine if the following violate the irrelevant alternatives criterion
PU

Number of Votes
Rank 14 13 16 15
1st B A C B
2nd C C A A
3rd A B B C

By pairwise comparison C won the election (verify), so the two irrelevant alternatives are A and B.

If A is eliminated, the preference table is now:

Number of votes 14 13 16 15
First place B C C B
Second place C B B C

All Rights Reserved. 2020 Abdul, Atienza, et. al.


Lesson 6 92

Now there are 14 + 15 = 29 voters who preferred B to C, and there are 13 + 16 =29 voters who
preferred C to B. The result is a tie , which shows that this election violates the irrelevant alternatives
criterion. Candidate C shouldn’t have gone from a win to a tie because one of the losing candidates
dropped out.

Fairness Criteria Satisfied by Various Voting Method


Head - to - Irrelevant
head Majority Monotonicity alternatives
criterion criterion criterion criterion
Plurality X X

S
Borda Count X

Plurality-with-elimination

Pairwise comparison

DM X
X

X X

Remark. In 1951, an economist named Kenneth Arrow was able to prove that there does not exist and
never will exist a democratic voting method for three or more alternatives that satisfies all four of the
fairness criteria. The result is know as Arrow’s impossibility theorem.

Approval Voting

In the late 1970s, a new voting method called the approval voting was introduced.
P
Approval Voting
With approval voting, each voter gives one vote to as many candidates on the ballot as he or
PU

she finds acceptable. The votes are counted and the winner is the candidate who receive the most
votes.

An election was held for an employee of the month award using the approval voting. The results are
shown in the table below. Which candidate won?
Number of votes 20 18 12 4
Candidate A X X
Candidate B X X X
Candidate C X X X
Candidate D X X
Candidate E X X

Solution. From the table, count the number of votes for each candidate.
Candidate A: 18+4=22 votes

All Rights Reserved. 2020 Abdul, Atienza, et. al.


Lesson 6 93

Candidate B: 20+12+4= 36 votes


Candidate C: 18+12+4=34 votes
Candidate D: 20+4=24 votes
Candidate E: 12+4=16 votes
Hence, in this election, candidate B wins with 36 votes.

Remark. In all voting methods , the possibility of a tie should be considered before the votes are counted
and some way of breaking a tie should be agreed on in advance of the election.

S
6.2 Apportionment
Let’s say you have 2 dozens donuts to be distributed among your 4 family members. Obviously, you can

DM
fairly distribute these 24 donuts among the 4 family members by giving 6 donuts for each person. This
is a very simple example of apportionment in the sense that everyone else in the group gets the same
number of donuts and there are no left over. But what if the number of family members does not evenly
divide the number of donuts? You may say that you can just cut the donuts into pieces. But what if
you are trying to distribute evenly computers to colleges in a university? machines to a hospital? cars
to a a police station? Can you still divide these items into smaller pieces?

This section is about coming up with ways fairly enough to divide and assign indivisible items. For that,
let us first consider the following terms.
P
Apportionment is a process of fairly assigning identical indivisible objects among individuals entitled
to shares that may be unequal.
PU

Seats are objects being assigned.

States or districts are individuals the seats are allocated to.

The term "indivisible" refers to objects that can’t be split into fractional parts, like people or seats in
Congress.

The standard divisor for an apportionment process is the average number if people per seat:

Total Population
Standard divisor =
Number of seats

The standard quota for a district in an apportionment process is the population of the district divided

All Rights Reserved. 2020 Abdul, Atienza, et. al.


Lesson 6 94

by the standard divisor:


Population of district
Standard quota =
Standard divisor

To calculate the upper quota, round the standard quota up to the next whole number.

To calculate the lower quota, round the standard quota down to the previous whole number.

Example 59. Find the upper and lower quotas for the following districts if 20 seats are to be allocated
proportionally among five districts according to the population (in thousands) of each district:

S
District 1 2 3 4 5 Total
Population 32 80 41 65 22 240
Standard Quota 2.67 6.67 3.42 5.42 1.83 20.01
Lower Quota
Upper Quota

DM 2
3
6
7
3
4
5
6
1
2
17
22

Remark. For a given apportionment problem, if it is just so happens that all of the standard quotas
are whole numbers, then we have a perfectly fair apportionment and all the methods (Hamilton’s,
Jefferson’s, Adam’s, Webster’s, Hunting-Hill) we will study in the next lessons will be unnecessary. But
in most real-life situations, it would not always be the case.
P
Hamilton’s Method

Hamilton’s Method
1. Calculate the standard divisor.
PU

2. Calculate the standard quota for each district.

3. Calculate the lower quota for each district.

4. Assign each district the number of seats according to its lower quota.

5. Assign any surplus of seats one by one to the districts in descending order of the fractional
parts of their standard quotas. The largest fractional part gets the first extra seat, then the
largest, and so on, until the correct number of seats is assigned.

Example 60. A large city with four districts plans to select 20 council members according to the
population (in thousands) of the districts shown below. Using Hamilton’s method, divide 20 seats
between the four districts.

All Rights Reserved. 2020 Abdul, Atienza, et. al.


Lesson 6 95

District 1 2 3 4 Total
Population 150 88 17 65 320

Solution.

District 1 2 3 4 Total
Population 150 88 17 65 320
Standard Quota 9.375 5.5 1.0625 4.0625
Lower Quota 9 5 1 4 19
Seats 9 6 1 4 20

S
Notice that the sum of the lower quotas is 19, which is 1 short of the number of seats. In this case, we
have one extra seat to assign and looking at the fractional parts of the standard quotas of each district,
0:5 is the largest, hence, District 2 gets the extra seat. Therefore, the seats are assigned as shown in
the last row of the table above.

Jefferson’s Method

Jefferson’s Method
1. Calculate the standard divisor.
DM
2. Calculate the standard quota for each district.

3. Calculate the lower quota for each district.


P
4. a. If the sum of the lower quotas is equal to the number of seats, assign each district the
number of seats equal to its lower quota.
b. If the sum of the lower quotas is less than the number of seats, choose a modified divisor
PU

by trial and error until the sum of the lower quotas equals the number of seats available.
The modified divisor will have to be less than the standard divisor in order to raise the
number of seats assigned. Assign each district the number of seats corresponding to
its modified lower quota.

Example 61. A city has 15 seats to divide among three districts according to the population (in
thousands) shown in the table. Find the appropriate modified divisor to be used in Jefferson’s method.

District 1 2 3 Total
Population 86 191 52 329

Solution. The standard divisor is 329 divided by 15, or 21.9. The standard and lower quotas are
summarized in the table:

All Rights Reserved. 2020 Abdul, Atienza, et. al.


Lesson 6 96

District 1 2 3 Total
Population 86 191 52 329
Standard Quota 3.93 8.72 2.37
Lower Quota 3 8 2 13

Since the number of seats is too few, try using a modified divisor. Subtracting 0.5 from 21.9, we get
21.4. The modified quotas are summarized below:

District 1 2 3 Total
Population 86 191 52 329
Modified Quota 4.02 8.93 2.43

S
Modified Lower Quota 4 8 2 14

This added one seat is still not enough, since we need 15. Try subtracting 0.5 again and use 20.9. The
new modified quotas are summarized below:

District
Population
DM
Modified Quota
Modified Lower Quota
1
86
4.11
4
191
9.14
9
2 3
52
2.49
2
Total
329

15

Since the total of the lower quotas is now the number of seats we’re looking for, we can stop here and
assign seats according to the last modified lower quota.

Adam’s Method
P
Adam’s Method
The Adam’s method is similar to Jefferson’s method, except that, it uses modified upper quotas
PU

rather than lower.

Example 62. Assign 15 seats from Example 14 using the Adam’s method. Does it result in a different
apportionment?

District 1 2 3 Total
Population 86 191 52 329

Solution. The standard divisor is 329 divided by 15, or 21.9. The standard and upper quotas are
summarized in the table:

District 1 2 3 Total
Population 86 191 52 329
Standard Quota 3.93 8.72 2.37
Upper Quota 4 9 3 16

All Rights Reserved. 2020 Abdul, Atienza, et. al.


Lesson 6 97

The upper quotas assign 16 seats, which is one more than the number of seats to be distributed. Since
we need fewer seats, we will increase the divisor in increments of 0.5 until the modified upper quotas
sum up to 15. Using 23.9 will have the following results:

District 1 2 3 Total
Population 86 191 52 329
Modified Quota 3.6 7.99 2.18
Modified Upper Quota 4 8 3 15

Notice that apportionment is in fact different from the result using the Jefferson’s method. The Jeffer-
son’s method is more likely to assign extra seats to larger districts while Adam’s favours smaller districts.

S
Webster’s Method

Adam’s Method

DM
1. Calculate the standard divisor.

2. Calculate the standard quota for each district.

3. Calculate the lower and upper quotas for each district.

4. To find the initial assignment, use standard rounding rules on the standard quota: round up
if the fractional part is 0:5 or greater, and round down if it is less than 0:5.
P
5. Check to see if the sum of the seats is equal to the number of seats available. If so, use use
the assignments from step 4. If not, use a modified divisor and reassign seats based on the
criteria in step 4, then repeat step 5. (If there are too many seats, try a larger divisor. If
there are too few seats, try a smaller divisor.)
PU

Example 63. Use Webster’s method to assign 27 seats according to population (in thousands) for the
following city with four districts.

District 1 2 3 4 Total
Population 149 83 92 126 450

Solution. The standard divisor is 450/27 =16.7. The corresponding standard quotas and initial assign-
ments of seats summarizes in the table.

District 1 2 3 4 Total
Population 149 83 92 126 450
Standard Quota 8.92 4.97 5.51 7.54
Initial Assignments 9 5 6 8 28

All Rights Reserved. 2020 Abdul, Atienza, et. al.


Lesson 6 98

We have one extra seat, so we should try a larger divisor. We’re pretty close, so we try a modified divisor,
say 16.8:

District 1 2 3 4 Total
Population 149 83 92 126 450
Standard Quota 8.87 4.94 5.48 7.5
Modified rounded quota 9 5 5 8 27

Now we can assign the seats according to the modified rounded quota.

S
Hunting-Hill Method

Hunting-Hill Method

DM
1. Calculate the standard divisor.

2. Calculate the standard, lower, and upper quotas and the geometric mean of the lower and
upper quotas for each district.

3. Initially assign the lower quota if the standard quota is less than the geometric mean of the
upper and the lower quotas; assign the upper quota if the standard quota is greater than
the geometric mean of the upper and lower quotas.

4. If the sum of the seats assigned in step 3 is equal to the number of seats available, leave
seats assigned according to step 3. If the sum of the seats assigned in step 3 is not equal to
P
the number of seats available, use a modified divisor as in the previous three methods and
reassign seats accordingly until the sum equals the total number of seats available.
PU

Example 64. A club with 400 members is to assign 10 chair positions to represent the members coming
from various communities . The distribution of the members is as follows:

Community A B C Total
Number of Members 153 75 172 400

Solution. The standard divisor is 400/10 = 40. The standard, upper, and lower quotas are given in the
table

Community A B C Total
Number of Members 153 75 172 400
Standard Quota 3.825 1.875 4.3
Lower Quota 3 1 4
Upper Quota 4 2 5

All Rights Reserved. 2020 Abdul, Atienza, et. al.


Lesson 6 99

Compute for the geometric means of the lower and upper quotas and compare each community’s stan-
dard quota to its geometric mean to decide whether to initially assign the lower quota or upper quota.

√ √
Community A: 3 · 4 = 12 ≈ 3:46. Choose the upper quota, 4.
√ √
Community B: 1 · 2 = 2 ≈ 1:41. Choose the upper quota, 2.
√ √
Community C: 4 · 5 = 20 ≈ 4:47. Choose the lower quota, 4.

Since the sum of the quotas assigned (4+2+4=10) already add up to the total number of seats, there
is no need to try a modified divisor.

S
Apportionment Flaws

Quota Rule

DM
Every district in an apportionment should be assigned either its upper quota or its lower quota.
An apportionment that violets the rule is said to folate quota.

Just as there are fairness criteria for voting methods, there are fairness criteria for apportionment methods.
In a fair method, none of the three paradoxes below should occur, and the method must satisfy the quota
rule. The question is, "Is there an apportionment method that is completely fair?"

• The Alabama Paradox


An increase in the total number of seats to be apportioned causes a district to lose a seat.
P
• The Population Paradox
An increase in a district can cause it to lose a slower growing districts.

• The New States Paradox


PU

Adding a state with its fair share of seats can affect the number of seats due other states.

To answer the question above, there is none. We have seen the the Hamilton’s method satisfies the
quota rule but may violate all of the three paradoxes. In addition, the other four apportionment methods
discussed all used modified divisors, hence, they can occasionally violate the quota rule. In fact, The
Balinski-Young Impossibility Theorem shows that any apportionment method which always follows
the quota rule will be subject to the possibility of paradoxes like the Alabama, New States, or Population
paradoxes. In other words, we can choose a method that avoids those paradoxes, but only if we are
willing to give up the guarantee of following the quota rule.

To summarize, there is no perfect method for either voting or apportionment and it is always left to the
discretion of the people involved to decide on a method that is most fair for their needs.

All Rights Reserved. 2020 Abdul, Atienza, et. al.


Lesson 6 100

Assessment
1. The students in Dr. Lee’s math class are asked to vote on the starting time for their final exam.
Their choices are M for 8:00AM, A for 10:00AM, T for 12NN, or H for 2:00PM. The results of
the election are shown in the preference table below.

Number of votes 8 12 5 3 2 2
First Choice M A T H A M
Second Choice A M H T T H

S
ThirdChoice T H A M M A
Fourth Choice H T M A H T

DM
(a) Determine the starting time using the plurality method.

(b) Using the plurality method, was the head-to-head comparison criterion violated?

(c) Determine the starting time using the Borda count method.

(d) Using the Borda count method, was the majority criterion violated?

(e) Determine the starting time using the plurality with elimination method.
P
(f) Using the plurality with elimination, was the majority criterion violated?
PU

(g) Determine the starting time using the pairwise comparison method.

(h) If a room for Dr. Lee’s final exam was not available at 2:00PM, and the votes were recounted
in the election, is the irrelevant alternatives criterion violated?

2. A large chain store needs to assign 12 buyers to its five stores. The number of employees for
each store is shown. Determine how many buyers should be assigned to each store using the (a)
Hamilton’s method and (b) Huntington-Hill’s method.

Store 1 2 3 4 5 Total
Employees 56 32 74 62 86

All Rights Reserved. 2020 Abdul, Atienza, et. al.


Final Exam in GEED 10053

Instructions. This is a multiple choice exam. Fully shade the circle corresponding to your answer. If your answer
is not among the choices, shade the circle corresponding to choice E. Avoid erasures and do not put unnecessary
markings on the answer sheet. Turn to the next page for the problems.
Final Exam in GEED 10053 (Mathematics in the Modern World) Page 1

1. Which of the following best describes the true nature of mathematics?


(a) Mathematics consists of doing arithmetic operations and calculations.
(b) Mathematics sheds light to the nature of reality.
(c) Mathematics is the study of patterns and relationships.
(d) Mathematics is the study of the universe.

2. By ignoring the colors, which of the following logos is a cyclic rosette pattern?

(a) (b) (c) (d)

3. If a frieze pattern only admits symmetries generated by one translation, one horizontal reflection, and one glide
reflection, which of the following classifies the pattern using the John B. Conway naming system?
(a) Jump (b) Hop (c) Siddle (d) Step

4. Which of the following equals the 10th Fibonacci number F10 ?


(a) 21 (b) 34 (c) 55 (d) 89

5. Leonardo Da Vinci named this ratio as the “divine proportion”. This ratio can be approximated by taking a
large positive integer n and divide Fn by Fn+1 . What is the name of this ratio?
(a) Holy Ratio (b) Bronze Ratio (c) Silver Ratio (d) Golden Ratio

6. If p and q are given true propositions, which of the following is also true?
(a) p −→ (¬ q) (b) (¬ p) ∨ (¬ q) (c) p ∧ (¬ q) (d) (¬ p) −→ (¬ q)

7. Consider the conditional statement “If a pentagon has less than five sides, then an icosahedron has at least
fifteen faces.” Which of the following gives the contrapositive of the conditional statement?
(a) An icosahedron has at least fifteen faces only if a pentagon has less than five sides.
(b) If an icosahedron has less than fifteen faces, then a pentagon has at least five sides.
(c) If an icosahedron has at least fifteen faces, then a pentagon has less than five sides.
(d) If a pentagon has at least five sides, then an icosahedron has less than fifteen faces.

8. Which of the following collections is well-defined?


(a) the collection of all stars in the universe
(b) the collection of all real numbers whose square is irrational
(c) the collection of all sets
(d) the collection of all mathematicians 12 feet tall

9. Which of the following sets is equivalent to the set A = {4; 5; 6; 7; 8}?


(a) {1; 2; 3; 4} (b) {x | x is prime and 1 < x < 12}
(c) {x ∈ R | 4 ≤ x ≤ 8} (d) {n ∈ N | n < 9}
Final Exam in GEED 10053 (Mathematics in the Modern World) Page 2

10. In a class of 50 students, sixty percent use neither an iPhone nor an iPad. Twenty percent use an iPhone, while
thirty percent use an iPad. How many students in this class are using both an iPhone and an iPad?
(a) 5 (b) 10 (c) 15 (d) 30

11. Which of the following is a type of reasoning where we make conjectures based on observable examples?
(a) inductive reasoning (b) deductive reasoning (c) hyperactive reasoning (d) selective reasoning

12. The conjecture “If n is a positive integer, then n2 + n + 1 is prime” is false. Which of the following values of n
is a counterexample?
(a) n = 3 (b) n = 5 (c) n = 2 (d) n = 4

13. Which of the following illustrates deductive reasoning?


(a) I’m going to be rich someday because everyone in my family who graduates in college got rich and I just
graduated college.
(b) On Christmas day, movie theaters and Chinese restaurants are always open. Therefore, this Christmas, we
can go to a movie and get some Chinese take out food.
(c) My teacher usually give surprise quizzes on Friday. Today is Thursday, so I must review my lessons because
my teacher might give a surprise quiz tomorrow.
(d) Note that 92 = 81, 992 = 9; 801, and 9992 = 998; 001. Therefore, 99992 = 99; 980; 001.

14. Using inductive reasoning, determine which of the following is the sum of

1 1 1 1
+ + + ··· + ?
1·2 2·3 3·4 99 · 100

49 50 99 100
(a) 50
(b) 51
(c) 100
(d) 101

15. A piece of rope is 48 inches long and is cut so that one piece is twice the as long as the other. Which of the
following is the length of the longer piece?
(a) 8 in (b) 16 in (c) 24 in (d) 32 in

16. Which of the following is the type of sampling where every possible subsets of size n from a population of size
N has the same chance of being selected?
(a) purposive sampling (b) simple random sampling
(c) quota sampling (d) non-probability sampling

17. The totality of elements that we are interested to study in a statistical investigation is called the
(a) population (b) sample (c) parameter (d) statistic

18. This level of measure classifies data into categories which can be ranked; however, the precise differences of
these categories may not be clear. Which one is it?
(a) ratio level (b) interval level (c) ordinal level (d) nominal level
Final Exam in GEED 10053 (Mathematics in the Modern World) Page 3

19. Which of the following measures of central tendencies is most appropriate if the data can be measured up to
the nominal level only?
(a) mean (b) median
(c) mode (d) none of the other choices

20. This measure of variability can be calculated by taking the average deviation from the mean. Which one is it?
(a) range (b) variance (c) standard deviation (d) mean

21. Vermae borrowed PhP 5,500 to her friend and agrees to repay it in 90 days with 5% annual rate. If the amount
of the interest is computed using the banker’s rule, how much does Vermae need to pay?
(a) PhP 5,567.81 (b) PhP 5,568.75 (c) PhP 5,567.80 (d) PhP 68.75

22. Andrew deposited half of PhP 130,000 in a certificate of deposit (CD) paying 4% interest, compounded
quarterly. How much will Andrew have in his account after 7 and a half years if he will not withdraw nor
deposit into this account?
(a) PhP 175,220.35 (b) PhP 87,610.17 (c) PhP 175,220.36 (d) PhP 87,610.18

23. Kenneth wants to have PhP 5,000,000 in his account when he retires in 25 years. The retirement account earns
5% interest convertible monthly. How much does he need to deposit each month to meet his retirement goal?
(a) PhP 29,229.51 (b) PhP 29,229.50 (c) PhP 8,396.17 (d) PhP 8,396.16

24. A loan of PhP 75,000 with level payments to be made at the end of every month for 24 months with monthly
rate of 3%. How much is the regular installment payment?
(a) PhP 4,428.56 (b) PhP 4,428.55 (c) PhP 3,223.59 (d) PhP 3,223.60

25. A loan of PhP 75,000 with level payments to be made at the end of every month for 24 months with monthly
rate of 6% along with deposits to a sinking fund that earns a monthly rate of 3%. How much is the amount
in the sinking fund after 3rd payment?
(a) PhP 132.67 (b) PhP 6,733.71 (c) PhP 6,535.68 (d) PhP 68,266.29

26. The members of Math Club are to choose a representative among four candidates. The following table shows
the result of the election.

Number of Votes
Rank 60 20 30
1st A B C
2nd B C D
3rd C D B
4th D A A

Using Borda Count method, determine who is the winner of the election. By using this method, was the
majority criterion violated?
(a) Candidate A is the winner, the majority criterion is violated.
(b) Candidate B is the winner, the majority criterion is violated.
(c) Candidate C is the winner, the majority criterion is not violated.
(d) Candidate D is the winner, the majority criterion is not violated.
Final Exam in GEED 10053 (Mathematics in the Modern World) Page 4

27. Consider the following preference table for the city that will host the Pambansang Palaro 2021. Using the
Plurality-with-Elimination Method, which city wins the election?

Number of Votes
Rank 7 8 10 4
1st Manila Pasig Marikina Manila
2nd Pasig Marikina Manila Marikina
3rd Marikina Manila Pasig Pasig

(a) Manila (b) Marikina


(c) Pasig (d) None of the other choices

28. The following statements are true except


(a) The plurality method of voting satisfies the majority criterion.
(b) The Borda count method of voting always violates the majority criterion.
(c) The plurality-with-elimination method of voting satisfies the majority criterion.
(d) The pairwise comparison method of voting satisfies the head-to-head criterion.

29. If the standard quota of a certain district is 4, then what is its lower quota?
(a) 3 (b) 4 (c) 5 (d) 6

30. A school needs to assign 35 leaders to represent the students from Luzon, Visayas and Mindanao. The
distribution of the population of students (in thousands) is as follows:

Luzon Visayas Mindanao Total


Students Population 27 18 3 48

Determine how many leaders should be assigned to represent students in Luzon, Visayas and Mindanao using
the Hamilton’s method.
(a) Luzon: 19, Visayas: 14, Mindanao: 2 (b) Luzon: 19, Visayas: 13, Mindanao: 3
(c) Luzon: 20, Visayas: 12, Mindanao: 3 (d) Luzon: 20, Visayas: 13, Mindanao: 2

You might also like